Download as pdf or txt
Download as pdf or txt
You are on page 1of 116

Contents

Contents

iv

List of Tables

List of Figures

vi

Pendahuluan

Mathematics Proof Method


Metode Pembuktian Matematis
1.1

Direct Proof
Bukti Langsung . . . . . . . . . . . . . . . . . . . . . . . . . . . . .

1.2

Indirect Proof
Bukti Tak Langsung . . . . . . . . . . . . . . . . . . . . . . . . . . .

Number Theory
Teori Bilangan
2.1

11

Divisibility
Keterbagian . . . . . . . . . . . . . . . . . . . . . . . . . . . . . . .

2.2

11

Special Number
Bilangan Khusus . . . . . . . . . . . . . . . . . . . . . . . . . . . .
2.2.1

14

A Prime and Composite Number


Bilangan Prima dan Komposit . . . . . . . . . . . . . . . .

14

CONTENTS
2.2.2

ii
Perfect Square
Bilangan Kuadrat Sempurna . . . . . . . . . . . . . . . . .

15

2.3

GCD dan Algoritma Euclid . . . . . . . . . . . . . . . . . . . . . .

17

2.4

Modular Arithmetic
Modulo Aritmatik . . . . . . . . . . . . . . . . . . . . . . . . . . . .

2.5

Linear Diophantine Equations


Persamaan Linier Diophantin . . . . . . . . . . . . . . . . . . . . .

Algebra Functions
Fungsi Aljabar
3.1

23

28

Polynomials Inequality
Pertidaksamaan Polinomial . . . . . . . . . . . . . . . . . . . . . .
3.1.1

3.1.2

28

Inverse Function
Fungsi Invers . . . . . . . . . . . . . . . . . . . . . . . . . .

32

Arithmetic and Geometric Sequence


Barisan Aritmatik dan Geometrik . . . . . . . . . . . . . .

3.2

19

34

Arithmetic, Geometric, Harmonic, and Quadratic Means


Rataan Aritmatik, Geometrik, Harmonik dan
Kuadratik . . . . . . . . . . . . . . . . . . . . . . . . . . . . . . . .

3.3

43

The Polynomials and Remainder Theorem


Suku Banyak dan Teorema Sisa . . . . . . . . . . . . . . . . . . . .
3.3.1

Polynomials Division
Pembagian Suku Banyak . . . . . . . . . . . . . . . . . . .

3.3.2

47

47

Remainder Theorem
Teorema Sisa . . . . . . . . . . . . . . . . . . . . . . . . . .

49

CONTENTS
3.3.3

iii
Factor Theorem
Teorema faktor . . . . . . . . . . . . . . . . . . . . . . . . .

3.3.4

Properties of Polynomial Roots


Sifat-Sifat Akar-Akar Suku Banyak . . . . . . . . . . . . . .

Trigonometry
Trigonometri
4.1

4.1.1

57

4.1.2

63

Formulas of Sum and Difference of Angles


Rumus-rumus Jumlah dan Selisih Sudut . . . . . . . . . .

4.1.3

58

Sine and Cosine Rule


Aturan Sinus dan Cosinus . . . . . . . . . . . . . . . . . . .

65

Trigonometric Equation
Persamaan Trigonometri . . . . . . . . . . . . . . . . . . . .

69

Limit Fungsi . . . . . . . . . . . . . . . . . . . . . . . . . . . . . . .

71

4.2.1

Solution Techniques
Metode Penyelesaian . . . . . . . . . . . . . . . . . . . . . .

4.2.2

4.2.3

75

Limit of Algebraic Function


Limit Fungsi Aljabar . . . . . . . . . . . . . . . . . . . . . .

76

Limit of Trigonometric Function


Limit Fungsi Trigonometri . . . . . . . . . . . . . . . . . . .

50

Trigonometric Function
Fungsi Trigonometri . . . . . . . . . . . . . . . . . . . . . . . . . .

4.2

49

79

Kombinatorika

86

5.1

86

Permutasi dan Kombinasi . . . . . . . . . . . . . . . . . . . . . . .

CONTENTS

iv

5.2

Prinsip Inklusi-Ekslusi dan Peluang . . . . . . . . . . . . . . . . .

87

5.3

Koefisien Binomial . . . . . . . . . . . . . . . . . . . . . . . . . . .

88

5.4

Prinsip Sarang Merpati . . . . . . . . . . . . . . . . . . . . . . . . .

92

5.5

Paritas . . . . . . . . . . . . . . . . . . . . . . . . . . . . . . . . . .

93

5.6

Relasi Rekurensi . . . . . . . . . . . . . . . . . . . . . . . . . . . . .

94

5.7

Soal-soal dan Pembahasan . . . . . . . . . . . . . . . . . . . . . . .

94

List of Tables
4.1

Trigonometric quadrant system . . . . . . . . . . . . . . . . . . . .

59

4.2

The value of trigonometric functions for special angles . . . . . .

60

4.3

The value of trigonometric functions for any angle (X o ) . . .

61

4.4

The value of trigonometric functions for negative angles . . . . .

62

4.5

The value of trigonometric functions for any angle (X o + ) . . .

62

List of Figures
3.1

Squares in the circles. . . . . . . . . . . . . . . . . . . . . . . . . . .

37

4.1

The right triangle trigonometric system . . . . . . . . . . . . . . .

58

4.2

Triangle and circle of radius R . . . . . . . . . . . . . . . . . . . . .

64

4.3

Sum and Difference of Angles . . . . . . . . . . . . . . . . . . . . .

66

vi

CHAPTER 1

Mathematics Proof Method


Metode Pembuktian Matematis
In mathematics, a proof is a convincing demonstration (within the accepted standards of a field study) to show that some mathematical statement is necessarily
true. Proofs are obtained from deductive reasoning, rather than from inductive
or empirical arguments. That is, a proof must demonstrate that a statement is
true in all cases, without a single exception.
Dalam matematika, bukti adalah suatu demonstarasi meyakinkan (mengikuti
beberapa standar yang diterima dari suatu bidang kajian tertentu) untuk menunjukkan bahwa pernyataan matematika itu betul-betul benar. Pembuktian
lebih diperoleh dari penarikan kesimpulan secara deduktif dibandingkan dengan penarikan kesimpulan yang secara induktif atau empiris. Dengan demikian,
bukti harus menunjukkan bahwa sebuah pernyataan itu adalah benar disegala
hal tanpa suatu perkecualian sedikitpun.
The statement that is proved is often called a theorem. Once a theorem is proved,
it can be used as the basis to prove further statements. A theorem may also be
referred to as a lemma, that is a sub theorem, especially if it is intended for use
as a stepping stone in the proof of another theorem. An implication of theorems
or lemmas is known as a corollary. An unproved proposition that is believed to
be true is known as a conjecture.
Sebuah pernyataan yang sudah terbuktikan disebut dengan teorema. Sekali teorema itu terbuktikan maka hal ini dapat digunakan sebagai dasar untuk membuktikan pernyataan-pernyataan selanjutnya. Kadangkala teorema disebut juga
1

Chapter 1. Mathematics Proof Method

dengan lemma, bagian kecil dari teorema, khusunya jika hal ini digunakan sebagai batu loncatan untuk membuktikan teorema-teorema lainnya. Sebuah akibat
dari beberapa teorema atau lemma disebut dengan korolary. Sebuah pernyataan
yang tidak terbuktikan namun diyakini kebenarannya dikenal dengan istilah
konjektur.
Proofs employ logic but usually include some amount of natural language which
usually admits some ambiguity. In fact, the vast majority of proofs in written mathematics can be considered as applications of rigorous informal logic.
Purely formal proofs, written in symbolic language instead of natural language,
are considered in proof theory. The distinction between formal and informal
proofs has led to much examination of current and historical mathematical practice, quasi-empiricism in mathematics, and so-called folk mathematics (in both
senses of that term). Therefore, the philosophy of mathematics is concerned with
the role of language and logic in proofs, and mathematics as a language.
Beberapa pembuktian matematika menggunakan logika, namun biasanya juga
melibatkan beberapa bahasa biasa yang kadangkala memunculkan dua arti. Namun demikian fakta menunjukkan bahwa hampir semua pembuktian dalam
pernyataan matematika dapat dikatakan sebagai suatu aplikasi dari logika matematika informal. Dalam pembuktian formal yang asli, penulisan dengan simbolsimbol matematika dibandingkan dengan penulisan dengan bahasa biasa lebih
dipilih dalam teori pembuktian. Perbedaan antara pembuktian formal dan informal telah mendasari beberapa evaluasi matematika dan sejarah latihan matematika akhir-akhir ini, termasuk juga matematika semi empiris, sehingga kita
mempunyai istilah yang dikenal dengan matematika untuk umum (terhadap
kedua istilah itu). Dengan demikian, filosofi matematika sesungguhnya adalah
terkait dengan bagaimana perananan bahasa dan logika dalam matematika itu
sendiri, sehingga matematika menjadi suatu bahasa.

Chapter 1. Mathematics Proof Method

Mathematics statement can be either true or false. A statement which is always


true is called a tautology, a statement which is always false is called a contradiction. To prove a truth of mathematics statement, we need a proof technique. Basically, there are two types of proof technique, namely direct proof and indirect
proof. In the following, we describe how the two techniques are implemented
in proving the truth of a statement.
Pernyataan matematika dapat bernilai benar atau salah. Suatu pernyataan yang
selalu bernilai benar disebut tautologi, sedangkan pernyataan yang selalu bernilai salah disebut kontradiksi. Untuk membuktikan kebenaran suatu pernyataan
matematika dibutuhkan suatu metode pembuktian. Pada prinsipnya terdapat
dua metode pembuktian, yaitu bukti langsung dan bukti tak langsung. Berikut
ini akan dijelaskan bagaimana kedua metode itu diterapkan untuk membuktikan kebenaran suatu pernyataan.

1.1

Direct Proof
Bukti Langsung

In this case, to prove a truth of mathematics statement is utilized a direct way


with a particular technique in direction of having a conclusion. In general, there
are three direct proofs, namely one way proof (implication), two ways proof
(biimplication/equivalence) and mathematics induction. Some examples of the
use of those methods are presented in the following.
Dalam hal ini, pembuktian dalam kebenaran matematika dibuktikan dengan
cara langsung dengan teknik-teknik tertentu sampai mencapai kesimpulan. Secara umum, terdapat tiga pembuktian langsung yaitu pembuktian satu arah
(implikasi), pembuktian dua arah (biimplikasi/ekuivalensi) dan induksi matematika.
Beberapa contoh penggunaan metode tersebut adalah sebagai berikut.

Chapter 1. Mathematics Proof Method

Lemma 1.1.1 If n is a natural number then 1 + 2 + 3 + + n = n2 (n + 1) for any n.


Lema 1.1.1 Jika n suatu bilangan asli maka 1 + 2 + 3 + + n =
setiap n.

n
(n
2

+ 1) untuk

Proof. Implication Proof: Suppose Un = n2 = Un+1 = (n + 1)2 = n2 + 2n +


1 = Un+1 Un = 2n + 1. Then we have
Bukti. Pembuktian Implikasi: Misal Un = n2 = Un+1 = (n + 1)2 = n2 + 2n +
1 = Un+1 Un = 2n + 1. Maka didapat
Un+1 Un = 2n + 1
Un Un1 = 2(n 1) + 1
..
.
U3 U2 = 2 2 + 1
U2 U1 = 2 1 + 1.
Sum up the n equations above, we get
Jumlahkan seluruh n persamaan di atas maka akan didapat
Un+1 U1 = 2(1 + 2 + + n) + (1 + 1 + + 1)
n2 + n
1 + 2 + + n =
2
Therefore
Dengan demikian 1 + 2 + + n = n2 (n + 1).

Lemma 1.1.2 Let a, b be two integer numbers and n be a positif integer. For any integer
n, n|a and n|b will give the same remainder if and only if n|(a b).
Lema 1.1.2 Diberikan dua bilangan bulat a, b dan bilangan bulat positip n. Untuk
sebarang bilangan bulat n, na dan nb akan mempunyai sisa yang sama jika dan hanya
jika n|(a b).

Chapter 1. Mathematics Proof Method

Proof. Equivalence Proof:


Bukti. Pembuktian Ekuivalensi:
(=)
Let s be a remainder of a and b divided by n, we have a = kn + s and b = jn + s
for 0 s n dan k, j I.
Misal sisa pembagian bilangan a dan b oleh n adalah s maka a = kn + s dan
b = jn + s, dengan 0 s n dan k, j I.
a b = (kn + s) (jn + s)
= (kn jn)
= (k j)n.
Since k, j I, k j = p where p is also integer, and a b = pn, it shows that
n|(a b).
Karena k, j I maka kj = p dimana p juga bilangan bulat, sehingga ab = pn,
yang artinya n|(a b).
(=)
Suppose that n|(a b). We will prove that a and b will give the same remainder
when they are divided by n. Let a = kn + s1 and b = jn + s2 for 0 s1 n and
0 s2 n, we will show that s1 = s2 .
Misal n|(a b). Akan dibuktikan bahwa a dan b akan mempunyai sisa yang
sama bila dibagi n. Misal a = kn + s1 dan b = jn + s2 untuk 0 s1 n dan
0 s2 n, maka akan ditunjukkan s1 = s2 .
a b = pn
a = b + pn
= (jn + s2 ) + pn
= (j + p)n + s2
= kn + s2 .

Chapter 1. Mathematics Proof Method

Since the remainder of n|a is single, we have s2 = s1


Karena sisa dari n|a adalah tunggal maka s2 = s1 .

Lemma 1.1.3 Prove that 3|(22n 1) for n 1.


Lema 1.1.3 Buktikan bahwa 3|(22n 1) untuk n 1.
Proof. Mathematics Induction. For n = 1 3|(22 1) 3|3 (true). Suppose
it is true for n = k, we have 3|(22k 1). Thus, is that true for n = k + 1?
Bukti. Induksi Matematika. Untuk n = 1 3|(22 1) 3|3 (benar). Misal
benar untuk n = k maka 3|(22k 1). Selanjutnya apakah benar untuk n = k + 1?
3|(22(k+1) 1) ?
3|(22k+2 1)
3|(22k 22 1)
3|(22k 22 22 + 3)
3|(22 (22k 1) + 3).
Since 3|22 (22k 1) and 3|3. It follows that 3|(22 (22k 1) + 3), hence it is also true
for n = k + 1.
Karena 3|22 (22k 1) dan 3|3 maka 3|(22 (22k 1) + 3). Sehingga hal ini juga benar
untuk n = k + 1.

1.2

Indirect Proof
Bukti Tak Langsung

In this proof technique, we do not start involving the existing facts in direction of having a conclusion. We start the prof even from the opposite facts. In

Chapter 1. Mathematics Proof Method

general, there are two indirect proofs, namely contradictive proof and contrapositive proof. Contradictive proof is also called reductio ad absurdum proof.
For instance, we will proof that A is true, we start the proof by assuming that A
is not true. Contrapositive proof is obtained by finding a contraposition of an
implication statement in logic math. For instance, we will proof that p q is
true, we start the proof by determining the contraposition of p (q r), i.e.
(q r) p. As we know, in logic math p (q r) = (q r) p. In
the following, we present how to use these methods.
Dalam teknik pembuktian ini, fakta-fakta yang ada tidak digunakan secara langsung untuk menuju pada kesimpulan. Pembuktia dimulai justru dari hal sebaliknya. Secara umum terdapat dua pembuktian tak langsung, yakni pembuktian kontradiksi dan pembuktian kontraposisi. Pembuktian kontradiksi disebut juga
pembuktian kemustahilan. Misal yang akan dibuktikan adalah benarnya pernyataan A, maka pembuktian dimulai dengan mengandaikan bahwa A adalah
salah. Pembuktian kontraposisi diperoleh dari menentukan kontraposisi dari
sebuah pernyataan implikasi dalam logika matematika. Misal akan dibuktikan
bahwa pernyataan p = (qr) benar, maka pembuktian diawali dengan menentukan kontraposisi dari p = (q r), yaitu (q r) = p. Seperti yang kita
ketahui, dalam logika matematika p = (q r) = (q r) = p. Berikut ini
kita jelaskan bagaimana menggunakan metode pembuktian ini.
Lemma 1.2.1 Prove that

2 is an irrational number.

Lema 1.2.1 Buktikan bahwa

2 adalah suatu bilangan irasional.

Proof. Contradictive Proof. Suppose that 2 is a rational number. We will

have 2 = ab , where ab is a simplified form. By squaring the two sides, we


obtain a2 = 2b2 . It follows that a2 is even which implies that a is even. Suppose
a = 2k , we have (2k)2 = 2b2 b2 = 2k 2 which implies that b is also even.

Chapter 1. Mathematics Proof Method


Since both a and b are even,

a
b

is not a simplified form any more, which is a

contradiction.
Bukti. Pembuktian kontradiksi. Misal
a
b

(dimana

a
adalah
b
2
2

2 adalah bilangan rasional maka

2=

bentuk yang paling sederhana). Kuadratkan kedua ruasnya

diperoleh a = 2b , sehingga a2 adalah bilangan genap dan a pasti genap. Misal


a = 2k (2k)2 = 2b2 b2 = 2k 2 yang berakibat b juga genap. Bila a dan b
adalah sama-sama genap maka

a
b

bukan bentuk yang paling sederhana lagi, ini

jelas kontradiktif.

Lemma 1.2.2 Prove that if m + n 73 then m 37 or n 37, for m, n I.


Lema 1.2.2 Buktikan bahwa jika m + n 73 maka m 37 atau n 37, untuk
m, n I.
Proof. Contrapositive Proof. Consider the statement as a logic expression p =
(q r), where p m + n 73, q m 37 and r n 37. The contraposition
of the statement is (q r) = p or ( q r) = p. Therefore, to prove
the truth of the statement above, we can start proving that if m < 37 and n < 37
then m + n < 73. Suppose the two any numbers are m 36 dan n 36, we have
m + n 36 + 36 m + n 72 m + n < 73. It completes the prove that if
m + n 73 then m 37 or n 37, for m, n I.
Bukti. Pembuktian kontraposisi. Misal pernyataan tersebut disajikan dalam
p = (q r), dimana p m + n 73, q m 37 dan r n 37. Kontraposisi
pernyataan tersebut adalah (q r) = p atau ( q r) = p. Sehingga
untuk membuktikan kebenaran pernyataan di atas dapat dimulai dengan membuktikan bahwa jika m < 37 dan n < 37 maka m + n < 73. Misal sebarang dua
bilangan itu adalah m 36 dan n 36, maka m + n 36 + 36 m + n
72 m + n < 73, terbukti. Sehingga terbukti pulalah kebenaran pernyataan
semula.

Chapter 1. Mathematics Proof Method

P ROBLEMS

AND

S OLUTIONS

S OAL - SOAL DAN P EMBAHASAN


1. Prove that (an + b)m = bm mod n.
Buktikan bahwa (an + b)m = bm mod n.
Solution. Direct Proof. To prove (an + b)m = bm mod n, we need to show
that there exists an integer k such that (an + b)m bm = kn.
Solusi. Bukti Langsung. Untuk membuktikan (an + b)m = bm mod n
perlu ditunjukkan bahwa terdapat bilangan bulat k sedemikian hingga
(an + b)m bm = kn.

(an + b) b

m1

(an) + m(an)

m1

b + ... + m(an)b

+b

bn

= (an)m + m(an)m1 b + ... + m(an)bm1 + bm bm


= (an)m + m(an)m1 b + ... + m(an)bm1

m m1
m1 m1
m1 m1
=
(a) n
+ m(a)
n
b + ... + m(a)b
b
n

Let k =

m m1

(a) n

m1 m1

+ m(a)

m1 m1

b + ... + m(a)b

, hence we get

(an + b)m bm = kn. Therefore (an + b)m = bm mod n.

m m1
m1 m1
m1 m1
Bila k = (a) n
+ m(a)
n
b + ... + m(a)b
b
, maka didapat
(an + b)m bm = kn. Sehingga (an + b)m = bm mod n.
2. If p is a prime number and p|a1 a2 ...an then p|ai for any 1 i n.
Jika p adalah bilangan prima dan p|a1 a2 ...an maka p|ai untuk sebarang 1
i n.
Solution. Mathematics Induction. Let P (k) be a representation statement. Step I: If p|a1 , then it is obvious that P |ai for 1 i 1. Step II:

Chapter 1. Mathematics Proof Method

10

Assume it is true for P (k), that is if p|a1 a2 ...ak then p|ai for any 1 i k.
We will show that P (k+1) is also true. Since p|a1 a2 ...ak ak+1 and p is a prime
number, it holds p|a1 a2 ...ak or p|a(k+1) . From the two possibilities, we have
p|ai for 1 i k + 1 as P (k) is true. It concludes that P (k + 1) is also true.
Solusi. Induksi Matematika. Misal P (k) adalah representasi dari pernyataan tersebut. Langkah I : Jika p|a1 , jelas bahwa P |ai untuk suatu 1
i 1. Langkah II : Misalkan p(k) benar, artinya jika p|a1 a2 ...ak maka p|ai
untuk suatu 1 i k. Akan dibuktikan bahwa P (k + 1) benar. Diketahui p|a1 a2 ...ak ak+1 , maka karena p bilangan prima berlaku p|a1 a2 ...ak atau
P |a(k+1) . Dari kedua kemungkinan ini dikombinasikan, karena P (k) bernilai benar, maka didapatkan untuk suatu 1 i k + 1. Jadi terbukti bahwa
P (k + 1) bernilai benar.
3. Prove that if xm is divisible by a prime p, then x is also divisible by p.
Bukatikan bahwa jika xm habis dibagi bilangan prima p, maka x habis
dibagi p.
Solution. Since p divides xm , we have p|xm or p|x.xm1 . Since p is a prime,
we have p|x or p|xm1 . Suppose p does not divide x, it follows p|xm1 or
p|x.xm2 . Repeat the process, we will be able to show that p|x.
Solusi. Karena p membagi habis xm , didapat p|xm atau p|x.xm1 . Karena
p adalah bilangan prima maka p|x atau p|xm1 . Andaikan p tidak habis
membagi x, maka p|xm1 atau p|x.xm2 . Jika proses diteruskan maka akan
didapatkan bahwa p|x.

CHAPTER 2

Number Theory
Teori Bilangan
2.1

Divisibility
Keterbagian

When 13 is divided by 5, it will give quotient 2 and remainder 3, denoted by


13
5

= 2+

3
5

or 13 = 2 5 + 3. In general, for any positive integers a and b there

exists a unique pair (q, r) of nonnegative integers such that b = q a + r and


0 < r < a. We say that q is the quotient and r is the remainder when b is divided
by a. If r = 0 then we say that b is divisible by a or a divides b, denoted by a| b.
If b is not divisible by a then we denote as a - b.
Jika 13 dibagi 5 maka hasil baginya 2 dan sisanya 3 dan ditulis:

13
5

= 2+

3
5

atau

13 = 25+3. Secara umum, apabila a bilangan bulat dan b bilangan bulat positif,
maka ada tepat satu bilangan bulat q dan r sedemikian hingga b = q a + r dan
0 < r < a. Dalam hal ini, q disebut hasil bagi dan r adalah sisa pembagian bila b
dibagi a. Jika r = 0 maka dikatakan b habis dibagi a dan ditulis a| b. Bila b tidak
habis dibagi a maka ditulis a - b.

Lemma 2.1.1 (1) If a|b then a|bc for any c I; (2) If a| b and b| c then a| c; (3) If ab| c
then a| c and b| c; (4) If a|b and b|a then a = b; (5) If a| b and a| c then a| (bx + cy) for
any integers x and y.

11

Chapter 2. Number Theory

12

Lema 2.1.1 (1) Jika a|b maka a|bc untuk sebarang c I; (2) jika a| b dan b| c maka a| c;
(3) jika ab| c maka a| c dan b| c; (4) jika a|b dan b|a maka a = b; (5) jika a| b dan a| c
maka a| (bx + cy) untuk sebarang bilangan bulat x dan y.
Proof. Property (1): If a| b then b = ka, and if b| c then c = lb = l(ka) = (kl)a.
It implies that a| c. Property (3): If a| b then b = ka bx = kxa, if a| c then
c = la cy = kya. We have bx + cy = (kx + ly)a, therefore a| (bx + cy).
Bukti. Sifat (1): a| b maka b = ka, dan b| c maka c = lb = l(ka) = (kl)a maka
a| c. Sifat (3): a| b maka b = ka bx = kxa, dan a| c maka c = la cy = kya.
Kemudian bx + cy = (kx + ly)a maka a| (bx + cy).

Lemma 2.1.2 A number a is divisible by 2n if the last n digit of the number is divisible
by 2n .

Lema 2.1.2 Suatu bilangan a habis dibagi 2n jika n angka terkhir dari bilangan tersebut
habis dibagi 2n .
Proof. Suppose n = 1, thus 2 divides a if the last digit of the number is divisible by 2. Let a = . . . a3 a2 a1 a0 , we have a = 10(. . . a3 a2 a1 ) + a0 . Since
2| 10(. . . a3 a2 a1 ), a0 must be divisible by 2 to have 2|a.
Bukti. Misal n = 1, berarti a habis dibagi 2 jika angka terakhir dari bilangan
tersebut habis dibadi 2. Misal a = . . . a3 , a2 a1 a0 maka a = 10(. . . a3 , a2 a1 ) + a0 .
Karena 2| 10(. . . a3 , a2 a1 ) a0 harus habis dibagi 2 untuk memperoleh 2|a.
Example. Is 173332 divisible by 8? Since 23 | 332, we have 8| 173332.
Contoh. Apakah 173332 habis dibagi oleh 8? Karena 23 | 332 maka 8| 173332.

Chapter 2. Number Theory

13

Example. Is 13 + 23 + + 1003 divisible by 7? False, as we have 13 + 23 + +


1003 = (1 + 2 + 3 + + 100)2 = (5050)2 = 25502500 and 7 - 25502500.
Contoh. Apakah 13 + 23 + + 1003 habis dibagi 7? Tidak, karena 13 + 23 + +
1003 = (1 + 2 + 3 + + 100)2 = (5050)2 = 25502500 dan 7 - 25502500.
Lemma 2.1.3 A number a = an an1 . . . a1 a0 is respectively divisible by 3,9 and 11 if
the sum of its digits satisfies respectively the following: (an +an1 +an2 + +a1 +a0 )
is divisible by 3; (an + an1 + an2 + + a1 + a0 ) is divisible by 9; and (an an1 +
an2 an3 + . . . ) is divisible by 11.
Lema 2.1.3 Suatu bilangan a = an an1 . . . a1 a0 berturut-turut habis dibagi 3,9 dan
11 jika jumlah angka-angkanya memenuhi masing-masing sifat berikut: (an + an1 +
an2 + + a1 + a0 ) habis dibagi 3; (an + an1 + an2 + + a1 + a0 ) habis dibagi 9;
dan (an an1 + an2 an3 + . . . ) habis dibagi 11.
Proof.
Bukti.
a = an an1 . . . a1 a0
= an 10n + an1 10n1 + . . . a1 101 + a0 100
= an (9 + 1)n + an1 (9 + 1)n1 + . . . a1 (9 + 1)1 + a0 (9 + 1)0
= an [9n + n 9n1 + + 9n] + an + an1 [9n1 + (n 1) 9n2 +
+ 9(n 1)] + an1 + + 9a1 + a1 + a0
= an [9n + n 9n1 + + 9n] + an1 [9n1 + (n 1) 9n2 + +
9(n 1)] + + 9a1 + an + an1 + + a1 + a0
= K(a) + Q(a)
Since (3 9)| K(a), it must be (3 9)| Q(a) to have (3 9)| a .
Karena (3 9)| K(a) maka haruslah (3 9)| Q(a) supaya (3 9)| a .

Chapter 2. Number Theory

2.2

14

Special Number
Bilangan Khusus

2.2.1

A Prime and Composite Number


Bilangan Prima dan Komposit

The integer p > 1 is called a prime number if there is no integer d with d > 1
and d 6= p such that d|p. Any integer n > 1 has at least one prime divisor. If n is
a prime, then that prime divisor is n itself. An integer n > 1 that is not a prime
is called composite.
Bilangan bulat p > 1 disebut sebuah bilangan prima jika tidak ada bilangan
bulat d dimana d > 1 dan d 6= p sedemikian hingga d|p. Setiap bilangan bulat
n > 1 mempunyai paling sedikit satu pembagi prima. Jika n adalah bilangan
prima, maka pembagi primanya adalah bilngan n itu sendiri. Sebuah bilangan
bulat n > 1 yang bukan bilngan prima disebut bilangan komposit.

Theorem 2.2.1 Eratosthenes. For any composite n, there exists a prime p such that

p| n and p n. In other words If there does not exist a prime p which divides n,

where p n, then n is a prime.


Teorema 2.2.1 Eratosthenes. Untuk setiap bilangan komposit n ada bilangan prima

p sehingga p| n dan p n. Dengan kata lain Jika tidak ada bilangan prima p yang

dapat membagi n dengan p n maka n adalah bilangan prima.


Example. Are 157 and 221 prime numbers?. The primes which are less than

157 are 2, 3, 5, 7, 11. Since none of them divides 157, then 157 is a prime. The

primes which are less than 221 are 2, 3, 5, 7, 11, 13. Since 13| 221 then 221 is a
composite number.

Chapter 2. Number Theory

15

Contoh. Apakah bilangan 157 dan 221 bilangan prima?. Bilangan-bilangan

prima yang lebih kecil dari 157 adalah 2, 3, 5, 7, 11. Karena tidak ada satupun
dari bilangan- bilangan prima itu yang dapat membagi 157, maka 157 merupakan bilangan prima. Kemudian bilangan - bilangan prima yang lebih kecil

dari 221 adalah 2, 3, 5, 7, 11, 13. Karena 13| 221 maka 221 merupakan bilangan
komposit.

2.2.2

Perfect Square
Bilangan Kuadrat Sempurna

Any perfect square satisfies the following three properties:


The possible ones of perfect square number is either 0, 1, 4, 5, 6, dan 9.
If 4 divides a perfect square then the remainder is either 0 or 1.
If p is a prime and p| x2 then p| z, where z = x2 /p.
Sebarang bilangan kuadrat sempurna memenuhi tiga sifat berikut ini:
Angka satuan yang mungkin untuk bilangan kuadrat sempurna adalah 0,
1, 4, 5, 6, atau 9.
Jika 4 membagi bilangan kuadrat sempurna maka sisanya 0 atau 1.
Jika p bilangan prima dan p| x2 maka p| z, dimana z = x2 /p.
Example. Obtain a perfect square whose digits are k, k + 1, k + 2, 3k, k + 3.
Contoh. Carilah suatu bilangan kuadrat sempurna yang angka-angkanya berturutturut adalah k, k + 1, k + 2, 3k, k + 3.

16

Chapter 2. Number Theory

Solution. The ones of the number is k +3, it follows that k can be either 1, 2, 3 or
6. Whilst the tens is 3k, it follows that k can be either 0, 1, 2 or 3. They imply that
the possible k is either 1, 2 or 3 which give perfect square numbers 12334, 23465
or 34596. Since the remainder of 12334 divided by 4 is 2, it gives that 12334 is
not a perfect square. The remainder of 23465 divided by 4 is 1 and 5|23465, but
5 - 4693, so that 23465 is not a perfect square. Now, 4|34596, and we have the
following
Solusi. Angka satuan bilangan kuadrat ini adalah k + 3 sehingga k yang mungkin adalah 1, 2, 3, 6. Sedangkan angka puluhannya adalah 3k maka k yang mungkin adalah 0, 1, 2, 3. Dari kedua kemungkinan ini diperoleh k yang mungkin
adalah 1, 2, 3, dengan demikian bilangan kuadrat yang mungkin adalah 12334,
23465, 34596. Karena 12334 dibagi 4 bersisa 2 maka 12334 bukan bilangan kuadrat.
Bilangan 23465 dibagi 4 bersisa 1 dan 5|23465, akan tetapi 5 - 4693 maka 23465
bukan bilangan kuadrat. Sekarang, bilangan 4|34596, dan berikut ini berlaku
2 | 34596
2 | 17298
3 | 8649
3 | 2883
31 | 961
31 | 31
Therefore, 34596 = 22 32 312 = 1862 which is a perfect square.
Sehingga 34596 = 22 32 312 = 1862 yang merupakan bilangan kuadrat.

17

Chapter 2. Number Theory

2.3

GCD dan Algoritma Euclid

Let a, b be any integers. An integer d satisfying d| a and d| b is called a common


divisor of a and b. The biggest (respectively, smallest) value of d is called Greater
Common Divisor/GCD, denoted by GCD(a, b)) (respectively, Least Common Multiple/LCM, denoted by LCM (a, b))). Several techniques have been developed to
obtain GCD or LCM, one of them is Euclidean algorithm.
Misal a dan b sembarang bilangan bulat. Bilangan bulat d yang memenuhi sifat
d| a dan d| b disebut faktor persekutuan dari a dan b. Nilai terbesar dari d disebut
faktor persekutuan terbesar atau FPB dan ditulis dengan F P B(a, b) sedangkan
nilai terkecil dari d disebut kelipatan persekutuan terkecil atau KPK dan ditulis
KP K(a, b). Beberapa tekni telah dikembangkan untuk menentukan FPB dan
KPK, salah satunya adalah algoritma Euclid.
Algorithm 2.3.1 Euclidean Algorithm. Given two integer numbers a and b with a >
b > 0, GCD(a, b) can be obtained by repeating the following algorithm:
a = q1 b + r1 ;

0 < r1 < b

b = q2 r1 + r2 ;

0 < r 2 < r1

r1 = q3 r2 + r3 ;
..
.

0 < r 3 < r2

rn2 = qn rn1 + rn ;

0 < rn < rn1

rn1 = qn+1 rn + 0
If rn is the last divisor of the division process which gives a remainder 0 then rn is
GCD(a, b).

Algoritma 2.3.1 Algoritma Euclide. Diberikan dua bilangan bulat a dan b dengan

18

Chapter 2. Number Theory


a > b > 0, maka GCD(a, b) bisa dicari dengan mengulang algoritma berikut:
a = q1 b + r1 ;

0 < r1 < b

b = q2 r1 + r2 ;

0 < r 2 < r1

r1 = q3 r2 + r3 ;
..
.

0 < r 3 < r2

rn2 = qn rn1 + rn ;

0 < rn < rn1

rn1 = qn+1 rn + 0
Jika rn merupakan pembagi terakhir dari pembagian di atas yang memberikan sisa 0
maka rn merupakan GCD(a, b).
Example. Determine GCD(4840, 1512). Using Euclidean Algorithm, the solution is the following:
Contoh. Tentukan GCD(4840, 1512). Dengan menggunakan Algoritma Euclid
maka solusinya adalah sebagai berikut:
4840 = 3 1512 + 304
1512 = 4 304 + 296
304 = 1 296 + 8
296 = 37 8 + 0
Therefore
Jadi GCD(4840, 1512) = 8.

Lemma 2.3.1 Any a|c and b|c imply ab|c if and only if GCD(a, b) = 1.

Lema 2.3.1 Sebarang a|c dan b|c berakibat ab|c jika dan hanya jika GCD(a, b) = 1.

Chapter 2. Number Theory

19

Example. 3|30 and 5|30 imply 3 5|30 as GCD(3, 5) = 1, however 2|30 and 6|30
imply 2 6 - 30 as GCD(2, 6) 6= 1.
Contoh. 3|30 dan 5|30 maka 3 5|30 karena GCD(3, 5) = 1, akan tetapi 2|30 dan
6|30 maka 2 6 - 30 karena GCD(2, 6) 6= 1.

2.4

Modular Arithmetic
Modulo Aritmatik

Let a, b, and m be integers, with m > 1. We say that a and b are congruent
modulo m, denoted by a b (mod m), if the remainder of a divided by m and b
divided by m are the same.
Diberikan bilangan bulat a, b dan m dimana m > 1. Bilangan a dikatakan kongruen dengan b modulo m, dituliskan dengan a b (mod m), jika sisa pembagian a oleh m dan b oleh m memberikan sisa yang sama.

Lemma 2.4.1 If a and b are congruent modulo m then m| (a b).

Lema 2.4.1 Jika a dan b kongruen modulo m maka m| (a b).


Proof. a b (mod m) = a = q1 m + r and b = q2 m + r. Hence a b = (q1 q2 )m,
it follows m| (a b).
Bukti. a b (mod m) = a = q1 m + r dan b = q2 m + r. Kemudian a b =
(q1 q2 )m sehingga m| (a b).

Chapter 2. Number Theory

20

Lemma 2.4.2 (1) a b (mod m), b a (mod m) are equivalent with ab 0 (mod m);
(2) if a b (mod m) and b c (mod m) then a c (mod m); (3) if a b (mod m)
and d|m then a b (mod d); (4) if a b (mod m) and c d (mod m) then ax + cy
bx + dy (mod m) and ac bd (mod m), for any x, y I.

Lema 2.4.2 (1) a b (mod m), b a (mod m) adalah setara dengan ab 0 (mod m)
; (2) jika a b (mod m) dan b c (mod m) maka a c (mod m); (3) jika a
b (mod m) dan d|m maka a b (mod d); (4) jika a b (mod m) dan c d (mod m)
maka ax + cy bx + dy (mod m) dan ac bd (mod m), untuk sebarang x, y I.
Proof. a b (mod m) m|(a b) and c d (mod m) m|(c d). From
Lemma 2.1.1, we have m|((ab)x+(cd)y) or m|((ax+cy)(bx+dy)), therefore
ax + cy bx + dy (mod m).
Bukti. a b (mod m) m|(a b) dan c d (mod m) m|(c d). Sesuai
Lema 2.1.1 didapat m|((a b)x + (c d)y) atau m|((ax + cy) (bx + dy)), sehingga
ax + cy bx + dy (mod m).

It follows from Lemma 2.4.2, if f (x) is a polynomial of integer coefficients and


a b (mod m) then f (a) f (b) (mod m).
Akibat dari Lemma 2.4.2, jika f (x) adalah suatu fungsi polinom dengan koefisienkoefisien bilangan bulat dan a b (mod m) maka f (a) f (b) (mod m).
Example. Prove that for any natural n, S = 2903n 803n +261n 464n is divisible
by 7 and 271. Furthermore, prove that 1897|S.
Contoh. Buktikan bahwa untuk setiap bilangan asli n maka S = 2903n 803n +
261n 464n habis dibagi 7 dan 271. Buktikan juga bahwa 1897|S.
Solution. Since 2903 803 (mod 7) and 464 261 (mod 7), as well as 2903
464 (mod 271) and 803 261 (mod 271), from Lemma 2.4.2(4) we must have 7|S

Chapter 2. Number Theory

21

and 271|S. Furthermore, since 1897 = 7 271 and GCD(7, 271) = 1, it follows
from Lemma 2.3.1 that 1897|S.
Solusi. Karena 2903 803 (mod 7) dan 464 261 (mod 7), demikian juga 2903
464 (mod 271) dan 803 261 (mod 271) maka sesuai dengan Lema 2.4.2(4) dipastikan bahwa 7|S dan 271|S. Selanjutnya karena 1897 = 7271 dan GCD(7, 271) =
1, maka sesuai Lema 2.3.1 terbukti 1897|S.

Lemma 2.4.3 (am + b)n bn (mod m)

Lema 2.4.3 (am + b)n bn (mod m)


Proof. The proof is the same with showing that there exists an integer k such
that (am + b)n bn = km.
Bukti. Pembuktian ini sama artinya dengan membuktikan ada bilangan bulat
k sehingga (am + b)n bn = km.
(am + b)n bn = (am)n + n(am)n1 b + + n(am)bn1 + bn bn
= {a(am)n1 + an(am)n2 + + an (b)n1 }m
= km.
2
Example. Determine the ones of 19971991 .
Contoh. Tentukan angka satuan bilangan 19971991 .
Solution. From Lemma 2.4.3, the solution is as follows:

22

Chapter 2. Number Theory

Solusi. Dengan menggunakan Lema 2.4.3 maka solusinya adalah sebagai berikut:
The ones (Angka satuan) 19971991 = The remainder (Sisa bagi) 19971991 oleh 10
= (199 10 + 7)1991 (mod 10)
= 71991 (mod 10)
= 74497+3 (mod 10)
= (74 )497 73 (mod 10)
= (2401)497 343 (mod 10)
= (240 10 + 1)497 (34 10 + 3) (mod 10)
= 1 3 (mod 10)
= 3 (mod 10)
Thereforre, the ones of 19971991 is 3.
Sehingga, angka satuan bilangan 19971991 adalah 3.
It is easy to see that powering numbers 0, 1, 2, . . . , 9 give the following ones/units:
0 => 0 1 type, 1 => 1 1 type, 2 => 2, 4, 8, 6 4 types, 3 => 3, 9, 7, 1 4
types, 4 => 4, 6 2 types, 5 => 5 1 type, 6 => 6 1 type, 7 => 7, 9, 3, 1
4 types, 8 => 8, 4, 2, 6 4 types, 9 => 9, 1 2 types.
Mudah dicermati bahwa perpangkatan bilangan 0, 1, 2, . . . , 9 akan menghasilkan
angka satuan berikut: 0 => 0 1 jenis, 1 => 1 1 jenis, 2 => 2, 4, 8, 6 4
jenis, 3 = 3, 9, 7, 1 4 jenis, 4 => 4, 6 2 jenis, 5 => 5 1 jenis, 6 => 6 1
jenis, 7 => 7, 9, 3, 1 4 jenis, 8 => 8, 4, 2, 6 4 jenis, 9 => 9, 1 2 jenis.
Therefore, finding the units of exponential number can be obtained by the following ways: Consider the example above. Determine the ones of 19971991 . Since
the ones of the base is 7, the ones of the exponential number contains 4 types,
namely 7,9,3,1. Thus 1991/4 = 4 497 + 3, it implies that 19971991 = 19974497+3
which follows the ones of 73 3. Therefore the ones of 19971991 is 3.

Chapter 2. Number Theory

23

Dengan demikian pencarian angka satuan dari perpangkatan bilangan dapat


dilakukan dengan cara berikut: Misal pada contoh soal di atas. Berapa angka
satuan bilangan 19971991 . Karena angka satuan bilangan dasar adalah angka 7
maka angka satuan perpangkatan bilangan ini akan meliputi 4 jenis yaitu 7,9,3,1.
Selanjutnya 1991/4 = 4 497 + 3 sehingga 19971991 = 19974497+3 yang berakibat
angka satuan dari 73 3. Dengan demikian angka satuan bilangan 19971991
adalah 3.
Example. Determine the ones of 345678292784383951 .
Contoh. Tentukan angka satuan bilangan 345678292784383951 .
Solution. Since the ones of the base is 4, the ones of the exponential number
contains 2 types, namely 4,6. Thus 383951/2 = 2 191975 + 1, it implies that
345678292784383951 = 3456782927842191975+1 which follows the ones of 41 4.
Therefore the ones of 345678292784383951 is 4.
Solusi. Karena angka satuan bilangan dasarnya adalah 4 maka angka satuan
perpangkatan bilangan ini akan meliputi 2 jenis yaitu 4,6. Selanjutnya tentukan
2|383951 = 2 191975 + 1 sehingga 345678292784383951 = 3456782927842191975+1
yang berakibat angka satuan dari 41 4. Dengan demikian angka satuan bilangan 345678292784383951 adalah 4.

2.5

Linear Diophantine Equations


Persamaan Linier Diophantin

An equation of the form ax + by = c where a, b, c are fixed integers and a, b are


all different from zero is called a linear Diophantine equation if the solutions x, y
respect to integers.

24

Chapter 2. Number Theory

Persamaan ax + by = c dengan a, b, c bilangan-bilangan bulat dan a, b duaduanya bukan nol disebut persamaan linier Diophantine jika penyelesaiannya
dicari untuk bilangan-bilangan bulat.

Theorem 2.5.1 Diophantine equation ax+by = c is solvable if and only if GCD(a, b)|c.
Teorema 2.5.1 Persamaan Diophantine ax + by = c mempunyai penyelesaian jika dan
hanya jika GCD(a, b)|c.
Proof. Let d = GCD(a, b) and d| c. We have d| c c = kd for any integers k.
Whilst d| GCD(a, b) am + bn = d for any m and n such that:
k(am + bn) = kd
a(km) + b(kn) = c,
it implies that x = mk dan y = nk
Bukti. Misal d = GCD(a, b) dan d| c, maka d| c c = kd untuk sebarang
bilangan bulat k. Sedangkan d| GCD(a, b) am + bn = d untuk sebarang
bilangan bulat m dan n sehingga:
k(am + bn) = kd
a(km) + b(kn) = c,
berarti x = mk dan y = nk

Theorem 2.5.2 If d = GCD(a, b) and x0 , y0 are the solutions of Diophantine equation


ax + by = c, then the general solutions are
b
a
x = x0 + k and y = y0 k; where k is an integer parameter.
d
d

25

Chapter 2. Number Theory

Teorema 2.5.2 Jika d = GCD(a, b) dan x0 , y0 merupakan penyelesaian persamaan


Diophantine ax + by = c, maka penyelelesaian umum persamaan tersebut adalah :
b
a
x = x0 + k dan y = y0 k; dengan k parameter bilangan bulat.
d
d
Example. Find the general solutions of Diophantine equation 738x + 621y = 45.
Contoh. Tentukan solusi umum dari persamaan Diophantine 738x + 621y = 45.
Solution. Finding GCD(738, 621) with Euclidean Algorithm
Solusi. Mencari GCD(738, 621) dengan Alogaritma Euclide
738 = 1 621 + 117
621 = 5 117 + 36
117 = 3 36 + 9
36 = 4 9 + 0.
we get GCD(738, 621) = 9. Since 9| 45, the equation is solvable. Consider the 9
as a linear combination of 738 and 621.
diperoleh GCD(738, 621) = 9. Karena 9| 45 maka persamaan di atas mempunyai
penyelesaian. Jadikan 9 sebagai kombinasi linear dari 738 dan 621.
9 = 117 3 36
= 117 3(621 5 117) = 3 621 + 16 117
= 3 621 + 16(738 621)
9 = 16 738 19 621
multiplying the two sides by 5, we get 45 = 80 738 45 621. It implies that
x0 = 80, y0 = 95. Therefore the general solutions are:

Chapter 2. Number Theory

26

Kalikan kedua ruas dengan 5, diperoleh 45 = 80 738 45 621, sehingga


didapat x0 = 80, y0 = 95. Dengan demikian penyelesaian umumnya adalah:
621
k = 80 + 69k
9
738
y = 95
k = 95 82k
9

x = 80 +

P ROBLEMS

AND

S OLUTIONS

S OAL - SOAL DAN P EMBAHASAN


1. A number A is the smallest natural number which is a product of the smallest three of prime number. Determine two numbers between 200 and 300
which gives the same prime factor with A.
Bilangan A adalah bilangan asli terkecil yang merupakan hasil kali dari 3
bilangan prima terkecil. Sebutkan dua buah bilangan di antara 200 dan
300 yang mempunyai faktor prima yang serupa dengan bilangan A.
Solution. A = 2.3.5 = 30, the desired numbers are 24 .3.5 = 240 and
2.32 .5 = 270
Solusi. A = 2.3.5 = 30, jadi bilangan yang dicari adalah 24 .3.5 = 240 dan
2.32 .5 = 270
2. P (n) is a multiplication of the digits of number n, and S(n) is summation
of its digits. Determine the ones of n satisfying P (n) + S(n) = n if n is a
number consisting two digits.
P (n) didefinisikan sebagai perkalian antara angka-angka bilangan n dan
S(n) adalah penjumlahan antara angka-angka bilangan n. Tentukan angka
satuan n yang memenuhi P (n) + S(n) = n jika n adalah bilangan yang
terdiri dari dua angka.

27

Chapter 2. Number Theory


Solution. Assume the number n is ab, we have a, b as a tens and ones.

Solution. Asumsikan bilangan tersebut adalah ab, maka kita mengartikan


a sebagai puluhan dan b sebagai satuan.
P (n) + S(n) = n
a.b + (a + b) = ab
a.b + (a + b) = 10a + b
ab = 9a = a9
Therefore, the ones of the number is 9
Dari sini didapatkan bahwa satuan bilangan n adalah 9
3. What is the remainder of 13 + 23 + 33 + ... + 1003 divided by 7?
Berapakah sisa pembagian 13 + 23 + 33 + ... + 1003 oleh 7?
Proof.
Bukti.
13 + 23 + 33 + ... + 1003 = (1 + 2 + 3 + ... + 100)3
= (5050)2
= (101x50)2 (mod 7)
= (101)2 x(50)2 (mod 7)
= (2x72 + 3)2 x(72 + 1)2 (mod 7)
= 32 x1 (mod 7)
= 9 (mod 7)
= 2 (mod 7)

CHAPTER 3

Algebra Functions
Fungsi Aljabar
3.1

Polynomials Inequality
Pertidaksamaan Polinomial

In this section, we will show how so solve inequalities. Solving an inequality


means finding all of its solutions. A solution of an inequality is a number which
when substituted for the variable makes the inequality a true statement.
Dalam bagian ini, akan dipelajari bagaimana menyelesaikan pertidaksamaan.
Menyelesaikan pertidaksamaan berarti mencari semua himpunnan penyelesaian. Penyelesaian dari pertidakasamaan adalah sebuah bilangan dimana pada
saat bilangan itu disubstitusikan maka pertidaksamaan menjadi pernyataan yang
benar.
Some properties related to inequality are as follows:
If a > b then a = b + k, for any k.
If a > b then a + c > b + c and a.c > b.c where c > 0.
If a > b and b > c then a > c.
If a > b and c > d then a + c > b + d.
If a > b > 0 then 1/a > 1/b, and if a/b > 0 then a.b > 0.
28

Chapter 3. Algebra Functions

29

Beberapa sifat-sifat terkait dengan pertidaksamaan adalah sebagai berikut:


Jika a > b maka a = b + k, untuk sebarang k.
Jika a > b maka a + c > b + c dan a.c > b.c dimana c > 0.
Jika a > b dan b > c maka a > c.
Jika a > b dan c > d maka a + c > b + d.
Jika a > b > 0 maka 1/a > 1/b, dan jika a/b > 0 maka a.b > 0.
In general, we present inequality polynomials as:
Secara umum pertidaksamaan fungsi dinyatakan sebagai
a0 xn + a1 xn1 + + an1 x + an 0
The following steps shows how to solve inequality:.
factorizing the polynomials, if it is not factorizible then consider whether
the function is definite or not.
Draw a number line and show the roots of polynomial equality on it.
Put sign + or on each interval of the number line respecting to the following rules:
Consider the coefficient of the highest order of the inequality.
If the coefficient of the the highest order is positive (respectively, or
negative) then the right side of line number interval is + (respectively,
or ). Hence, place an alternate sign on the remaining interval.
Consider the sign of the inequality and confirm with the signs on the
interval to conclude the solution set.

Chapter 3. Algebra Functions

30

Lagkah-langkah berikut menunjukkan bagaimana cara menyelesaikan persamaan


ini.
Lakukan memfaktorkan terhadap fungsi polinomial, bila tidak dapat difaktorkan pertimbangkan apakah fungsi tersebut definit.
Gambar sebuah garis bilangan dan letakkan semua akar-akar persamaan
polinomial dalam garis bilangan.
Berikan tanda + atau pada setiap interval dengan prinsip berikut:
Lihat koefisien pangkat tinggi pertidaksamaan tersebut.
Jika koefisien pangkat tertingginya positif maka interval paling kanan
adalah +, bila negatif maka tulis dan kemudian interval selanjutnya
secara bergantian berlawanan tanda.
Lihat tanda pertidaksamaannya dan cocokkan dengan tanda tanda
dalam interval untuk menentukan himpunan penyelesaiannya.
Example. Determine x satisfying inequality x2 2x 80 0 and
Contoh. Tentukan nilai-nilai x yang memenuhi kedua pertidaksamaan x2 2x
80 0 dan
(2x 5)501 (x2 3x 4)(x2 + 7x 80)(x3 + 4x2 x + 1)2000
0
(x + 1)(x + 14)51
Solution. To solve this inequality we should consider discriminant D = b2 4ac
of quadratic function ax2 + bx + c = 0. If D < 0 and a > 0 then the quadratic
function is positive definite, If D < 0 and a < 0 the quadratic function is negative
definite. Furthermore, all number of even power must be positive.
Jika

31

Chapter 3. Algebra Functions

Solusi. Untuk menyelesaikan pertidaksamaan ini maka kita harus ingat konsep Deskriminan D = b2 4ac untuk fungsi kuadratik ax2 + bx + c = 0. Bila
D < 0 dan a > 0 maka fungsi kuadratik adalah definit positif, Bila D < 0 dan
a < 0 maka fungsi kuadratik adalah definit negatif. Kemudian setiap bilangan
yang berpangkat genap pasti bernilai positif.
x2 2x 80 0
(x 10)(x + 8) 0 Hp1 = {x| 8 x 10}
Whilst (Sedangkan)
(2x 5)501 (x2 3x 4)(x2 + 7x 80)(x3 + 4x2 x + 1)2000
0
(x + 1)(x + 14)51
(2x 5)501 (x 4)
0
(x + 14)51
For (Untuk) x 6= 14, (2x 5)501 (x 4)(x + 14)51 0
For (Untuk) x 6= 14, (2x 5)501 (x 4)(x + 14)51 0
|
{z
}

501

(x 4)(x + 14)51 0
5
Therefore (Sehingga) Hp2 = {x| 14 < x x 4}
2
It follows (Berarti) (2x 5)

Since the problem shows a conjunction relation then both solution sets Hp1 and
Hp2 must be joined. It implies that Hp = {x| 8 x

5
2

4 x 10}.

Karena masalah di atas menunjukkan relasi konjungsi, maka kedua himpunan


penyelesaian Hp1 dan Hp2 harus digabung. Sehingga diperoleh Hp = {x| 8
x

5
2

4 x 10}.

Example. The value of x satisfying inequality (x 2)4 (x2 x 10)|x2 x


1|(x + 1)2 1 is . . .
Contoh. Harga x yang memenuhi pertidaksamaan (x 2)4 (x2 x 10)|x2
x 1|(x + 1)2 1 adalah . . .

Chapter 3. Algebra Functions

32

(A) x 1 0 x 1 x 2
(B) 12 x 0

3
2

x2

(C) 1 x 0 1 x 2
(D) 12 < x

3
2

(E) All x (Semua harga x)

3.1.1

Inverse Function
Fungsi Invers

If f is a function from A to B then an inverse function for f is a function in the


opposite direction, from B to A, with the property that a round trip (a composition) from A to B to A (or from B to A to B) returns each element of the initial
set to itself. Thus, if an input x into the function produces an output y, then
inputting y into the inverse function f 1 (read f inverse, not to be confused with
exponentiation) produces the output x. Not every function has an inverse; those
that are called invertible.
Jika f adalah fungsi dari A ke B maka fungsi invers untuk f adalah suatu fungsi
balikan yang berlawanana arah, yaitu dari B ke A, dengan sifat rute balikan
itu (komposisi fungsi) adalah dari A ke B ke A (atau dari B ke A ke B) akan
mengembalikan masing-masing element dari himpunan asal ke elemen-element
itu sendiri. Dengan kata lain, jika dimasukkan x ke dalam fungsi memberikan
hasil y, maka dengan memasukkan nilai y ke dalam fungsi invers f 1 (baca f invers, biar tidak dibingungkan dengan pengertian eksponensial) akan diperoleh
hasil x. Tidak setiap fungsi memiliki balikan, fungsi yang tidak mempunyai
fungsi invers ini disebut invertibel.

33

Chapter 3. Algebra Functions

Theorem 3.1.1 If f (x) is function with x < then inverse function f 1 (x) satisfies
the following:

Teorema 3.1.1 Jika f (x) suatu fungsi dalam x < maka fungsi invers f 1 (x) memenuhi
hal berikut:
1. f f 1 (x) = x (f g)1 (x) = g 1 f 1 (x)
2. f (x) = ax + b = f 1 (x) =
3. f (x) =

ax+b
cx+d

= f 1 (x) =

4. f (x) = ax + bx + c =
5. f (x) =

xb
a

dx+b
cxa

1
f12
(x)

ax + b = f 1 (x) =

b
2a

q
1
a

D
4a

x2 b
a

6. f (x) = ax + b f g = px + q = g(x) =
7. f (x) = abx = f 1 (x) =

x+

px+qb
a

a log x

Bukti. For no. 4. From (1), we know that f f 1 (x) = x, so that for f (x) =
ax2 + bx + c we get
Proof. Untuk nomor 4. Dari (1) dipahami bahwa f f 1 (x) = x sehingga untuk
f (x) = ax2 + bx + c diperoleh
f (f 1 ) = x
a(f 1 )2 + b(f 1 ) + c = x
a(f 1 )2 + b(f 1 ) + c x = 0
1
f12

b
=
2a

1
b2 4ac
x+
a
4a
2

Chapter 3. Algebra Functions

3.1.2

34

Arithmetic and Geometric Sequence


Barisan Aritmatik dan Geometrik

In mathematics, an arithmetic sequence or arithmetic progression is a sequence


of numbers such that the difference of any two successive members of the sequence is a constant. For instance, the sequence 3, 5, 7, 9, 11, 13, . . . is an arithmetic progression with common difference 2. The sum of the terms of a arithmetic progression is known as a arithmetic series. Thus, the general form of a
arithmetic sequence is a, a + b, a + 2b, . . . , a + (n 1)b; and that of a geometric
series is (a) + (a + b) + (a + 2b) + + (a + (n 1)b).
Dalam matematika, suatu barisan aritmatika atau urutan aritmatika didefinisikan sebagai barisan bilangan sedemikian hingga beda dari dua bilangan yang
berurutan dari barisan itu adalah konstan. Sebagai contoh, barisan bilangan
3, 5, 7, 9, 11, 13, . . . adalah barisan aritmatika dengan beda 2. Jumlah suku-suku
dari barisan arimatika disebut dengan deret aritimatika. Bentuk umum dari
barisan aritmatika adalah a, a + b, a + 2b, . . . , a + (n 1)b; sedangkan bentuk
umum dari deret aritmatika adalah (a) + (a + b) + (a + 2b) + + (a + (n 1)b).
Meanwhile, A geometric sequence is a sequence of numbers where each term
after the first is found by multiplying the previous one by a fixed non-zero number called the common ratio. For example, the sequence 2, 6, 18, 54,. . . is a
geometric progression with common ratio 3. Similarly 10, 5, 2.5, 1.25, . . . is a
geometric sequence with common ratio 1/2. The sum of the terms of a geometric progression is known as a geometric series. Thus, the general form of
a geometric sequence is a, ar, ar2 , ar3 , ar4 , . . .; and that of a geometric series is
a + ar + ar2 + ar3 + ar4 + . . .
Sementara, suatu barisan geometrik adalah barisan bilangan dimana masingmasing suku setelah bilangan pertama didapat dari mengalikan bilangan se-

Chapter 3. Algebra Functions

35

belumnya dengan bilangan konstan yang tidak nol, yang disebut dengan rasio dan rasio ini selalu sama. Sebagai contoh, barisan bilangan 2, 6, 18, 54,. . .
adalah barisan geometrik dengan rasio 3. Sama hanya dengan barisan 10, 5,
2.5, 1.25, . . . adalah barisan geometrik dengan rasio 1/2. Jumlah suku-suku
dari barisan geometrik ini disebut dengan deret geometrik. Bentuk umum dari
barisan geometri adalah a, ar, ar2 , ar3 , ar4 , . . .; sedangkan bentuk umum dari
deret geometri adalah a + ar + ar2 + ar3 + ar4 + . . .

Lemma 3.1.1 Given an arithmetic sequence a+(a+b)+(a+2b)+ +(a+(n1)b),


where a is an initial term an b is a deference of any two successive terms. Then, we have
the followings:

Lema 3.1.1 Diberikan suatu deret aritmatika a+(a+b)+(a+2b)+ +(a+(n1)b),


dimana a adalah nilai suku awal dan b adalah beda suku-suku yang berurutan, maka
beberapa hal berikut berlaku:
Un = a + (n 1)b
n
Sn =
[U1 + Un ]
2
n
Sn =
[2a + (n 1)b]
2
1
Ut = a + (n 1)b
2

(3.1)
(3.2)
(3.3)
(3.4)

Lemma 3.1.2 If we insert some k numbers on between any two successive numbers of
arithmetic sequence and they form a new arithmetic sequence, then

36

Chapter 3. Algebra Functions

Lema 3.1.2 Jika diantara dua buah bilangan yang berurutan dari suatu deret aritmatika disisipi k buah bilangan dan membentuk deret aritmatika baru maka
b
k+1
= n + (n 1)k

b0 =

(3.5)

n0

(3.6)

Lemma 3.1.3 Let a + ar + ar2 + + arn1 be a geometric sequence, where a, r are


respectively an initial value and ratio. The followings hold for this squence
Lema 3.1.3 Diberikan suatu deret geometrik a + ar + ar2 + + arn1 , dimana a
adalah nilai suku awal dan r adalah rasio suku-suku yang berurutan. Beberapa hal
berikut berlaku untuk deret ini.
Un = arn1
a(rn 1)
Sn =
, r>1
r1
a(1 rn )
Sn =
, r<1
1 r
Ut = a rn1

(3.7)
(3.8)
(3.9)
(3.10)

Lemma 3.1.4 If we insert some k numbers on between any two successive numbers of
geometric sequence and they form a new geometric sequence, then
Lema 3.1.4 Jika diantara dua buah bilangan yang berurutan dari suatu deret geometrik
disisipi k buah bilangan dan membentuk deret geometrik baru maka

r0 = k+1 r
(3.11)
n0 = n + (n 1)k

(3.12)

37

Chapter 3. Algebra Functions

Lemma 3.1.5 For arithmetic and geometric series satisfy Un = Sn S(n 1).

Lema 3.1.5 Untuk deret aritmatik maupun geometrik berlaku Un = Sn S(n 1).

Lemma 3.1.6 An infinite convergence geometric series, where |r| < 1, satisfies S =
a
.
1r

Lema 3.1.6 Untuk deret geometrik tak hingga konvergen, dimana |r| < 1, berlaku
a
S = 1r
.
Example. Determine the sum of all radius of circles whose numbers are infinity.
Example. Tentukan jumlah seluruh jari-jari lingkaran berikut ini sampai pada
banyaknya lingkaran tak higga.

Figure 3.1: Squares in the circles.

38

Chapter 3. Algebra Functions

Furthermore, how to find the n-term of sequence 1, 4, 11, 22, 37, . . . . Both formulas Un can not be used to answer this problems as the sequence does not have
a common difference at first layer, namely 3,7,11,15. Thus then, this sequence
admits an arithmetic sequence of common difference 4. The solution can be obtained by considering polynomial f (x) = ak xk + ak1 xk1 + ak2 xk2 + + a0 x0 .
Selanjutnya, bagaimana menentukan suku ken dari barisan bilangan: 1, 4, 11,
22, 37, . . . . Kedua rumus Un di atas tidak dapat dipakai untuk menjawab pertanyaan ini, sebab beda pada layer pertama tidak sama, yaitu 3,7,11,15, baru
kemudian barisan beda ini merupakan barisan aritmatika dengan beda 4. Solusinya diperoleh dari mempertimbangkan fungsi polinomial f (x) = ak xk +
ak1 xk1 + ak2 xk2 + + a0 x0 .

Lemma 3.1.7 Let f (x) = a1 x + a0 ; x = 1, 2, . . . , n be a linear function series of the


polynomial, it has common difference b = a1 at firs layer.
Lema 3.1.7 Misal deret fungsi linear dari polinomial di atas adalah f (x) = a1 x +
a0 ; x = 1, 2, . . . , n, maka deret ini mempunyai beda b = a1 pada layer pertama.
Proof. An expansion of the linear function for the n-terms is as follows:
Bukti. Ekspansi fungsi linear sampai suku ken adalah sebagai berikut:
a1 + a0 , 2a1 + a0 , . . . , a1 (n 3) + a0 , a1 (n 2) + a0 , a1 (n 1) + a0 , a1 n + a0
hence the common difference of the successive terms is b = f (n) f (n 1) =
f (n 1) f (n 2) = = f (2) f (1) = a1 , namely
sehingga beda dari suku-suku yang berurutan adalah b = f (n) f (n 1) =
f (n 1) f (n 2) = = f (2) f (1) = a1 , yaitu
a1 + a0 , 2a1 + a0 , . . . , a1 (n 3) + a0 , a1 (n 2) + a0 , a1 (n 1) + a0 , a1 n + a0
|
{z
}
|
{z
} |
{z
}
a1

a1

a1

Chapter 3. Algebra Functions

39

Lema 3.1.8 Let f (x) = a2 x2 + a1 x + a0 ; x = 1, 2, . . . , n be a quadratic function series


of the polynomial, it has common difference b = 2a2 at second layer.

Lema 3.1.9 Misal deret fungsi kuadrat dari polinomial di atas adalah f (x) = a2 x2 +
a1 x + a0 ; x = 1, 2, . . . , n, maka deret ini mempunyai beda b = 2a2 pada layer kedua.
Proof. An expansion of the quadratic function for the n-terms is as follows:
Bukti. Ekspansi fungsi quadratic untuk suku ken adalah sebagai berikut:
a2 + a1 + a0 , 4a2 + 2a1 + a0 , . . . , a2 (n 1)2 + a1 (n 1) + a0 , a2 n2 + a1 n + a0
hence the differences of the successive terms are as follows:
sehingga beda layer pertama dari suku-suku yang berurutan adalah sebagai
berikut:
bn = f (n) f (n 1) = 2a2 n a2 + a1 = (2n 1)a2 + a1
bn1 = f (n 1) f (n 2) = 2a2 n 3a2 + a1 = (2n 3)a2 + a1
bn2 = f (n 2) f (n 3) = 2a2 n 5a2 + a1 = (2n 5)a2 + a1
..
.
b3 = f (3) f (2) = 5a2 + a1
b2 = f (2) f (1) = 3a2 + a1 .
Therefore, the common difference of the successive terms of the difference sequence of the first layer is
Dengan demikian, beda dari suku-suku yang berurutan pada barisan beda untuk layer pertama adalah
b = bn bn1 = bn bn1 = = b3 b2 = 2a2

40

Chapter 3. Algebra Functions


or
atau

a + a1 + a0 , 4a2 + 2a1 + a0 , 9a2 + 3a1 + a0 , 16a2 + 4a1 + a0 , 25a2 + 5a1 + a0


|2
{z
}|
{z
}|
{z
}|
{z
}
3a2 +a1

{z
2a2

5a2 +a1

}|

{z
2a2

7a2 +a1

}|

{z

9a2 +a1 Layer I

2a2 Layer II

Lema 3.1.10 Let f (x) = a3 x3 + a2 x2 + a1 x + a0 ; x = 1, 2, . . . , n, be a cubic function


series of the polynomial, it has common difference b = 6a3 at third layer.

Lema 3.1.11 Misal deret fungsi kubik dari polinomial di atas adalah f (x) = a3 x3 +
a2 x2 + a1 x + a0 ; x = 1, 2, . . . , n, maka deret ini mempunyai beda b = 6a3 pada layer
ketiga.
Proof. As exercise.
Bukti. Sebagai latihan.

We conclude inductively in the following theorem:


Secara induktif dapat disimpulkan dalam teorema berikut:
Theorem 3.1.2 The series of polynomial function f (x) = ak xk +ak1 xk1 +ak2 xk2 +
+ a0 x0 ; x = 1, 2, . . . , n has a common difference b = k!ak at layer k.

Teorema 3.1.2 Deret fungsi polinomial f (x) = ak xk + ak1 xk1 + ak2 xk2 + +
a0 x0 ; x = 1, 2, . . . , n, mempunyai beda yang sama b = k!ak pada layer kek.

41

Chapter 3. Algebra Functions


Proof. As exercise.
Bukti. Sebagai latihan.

Example. Given a series 15 + 32 + 63 + 108 + 167 + . . . . Obtain the nterm and


the nsum.
Example. Diberikan suatu deret 15 + 32 + 63 + 108 + 167 + . . . . Tentukan suku
ken dan jumlah ken.
Solution. The series above forms the following
Solusi. Deret di atas akan mengikuti pola berikut
15
+ 32} |+{z63} +
108} +
167} + . . .
| {z
| {z
| {z
17

31

45

59

| {z } | {z } | {z }
14

14

14

Since the series has a common difference at second layer, we have k = 2 and
the function f (x) = a2 x2 + a1 x + a0 . We need to find the coefficients a2 , a1 , a0 .
Theorem 3.1.2 implies b = k!ak = 2!a2 = 14 a2 = 7, it follows f (x) = 7x2 +
a1 x + a0 . Let Un = f (n) and Un = 7n2 + a1 n + a0 . We have the following:
Karena deret fungsi mempunyai beda yang sama pada layer kedua maka k = 2
dan rumus f (x) = a2 x2 + a1 x + a0 . Selanjutnya perlu dicari koefisien a2 , a1 , a0 .
Dari Theorema 3.1.2 diperoleh b = k!ak = 2!a2 = 14 a2 = 7. Sehingga
f (x) = 7x2 + a1 x + a0 . Misal Un = f (n) dan Un = 7n2 + a1 n + a0 , maka akan
didapat:
n = 1 7 + a1 + a0 = 15 a1 + a0 = 8
n = 2 28 + 2a1 + a0 = 32 2a1 + a0 = 4
From both equations, we have a1 = 4 dan a0 = 12 which implies that the
nterm is Un = 7n2 4n + 12.

42

Chapter 3. Algebra Functions

Dari kedua persamaan diperoleh nilai a1 = 4 dan a0 = 12. Sehingga suku


ken adalah Un = 7n2 4n + 12.
To find the nsum of the above series, we consider the following series
Untuk menentukan jumlah ken dari deret di atas, maka kita lihat deret berikut
Un 15, 32, 63, 108, 167, . . .
Sn 15, 47, 110, 218, 385, . . .
15, 47 , 110 , 218 , 385 + . . .
| {z } | {z } | {z } | {z }
32

63

108

167

| {z } | {z } | {z }
31

45

59

| {z } | {z }
14

14

Since the sequence of function has a common difference on the third layer, we
have k = 3 and function f (x) = a3 x3 + a2 x2 + a1 x + a0 . We need to find the
coefficients a3 , a2 , a1 , a0 . Theorem 3.1.2 implies b = k!ak = 3!a3 = 14 a3 = 73 .
Hence f (x) = 37 x3 + a2 x2 + a1 x + a0 . Let Sn = f (n) and Sn = 37 n3 + a2 n2 + a1 n + a0 .
We have the following:
Karena deret fungsi mempunyai beda yang sama pada layer ketiga maka k = 3
dan rumus f (x) = a3 x3 + a2 x2 + a1 x + a0 . Selanjutnya perlu dicari koefisien
a3 , a2 , a1 , a0 . Dari Theorema 3.1.2 diperoleh b = k!ak = 3!a3 = 14 a3 =

7
.
3

Sehingga f (x) = 37 x3 + a2 x2 + a1 x + a0 . Misal Sn = f (n) maka Sn = 73 n3 + a2 n2 +


a1 n + a0 , maka akan didapat:
7
38
+ a2 + a1 + a0 = 15 a2 + a1 + a0 =
3
3
56
85
n=2
+ 4a2 + 2a1 + a0 = 47 4a2 + 2a1 + a0 =
3
3
189
141
n=3
+ 9a2 + 3a1 + a0 = 110 9a2 + 3a1 + a0 =
3
3
n=1

Solving the three equations, we have a2 = 23 , a1 =


the nsum is Sn = 37 n3 + 32 n2 +

67
n.
6

67
6

and a0 = 0 which implies

Chapter 3. Algebra Functions

43

Dengan menyelesaikan ketiga persamaan di atas diperoleh nilai a2 = 32 , a1 =

67
6

dan a0 = 0. Sehingga jumlah suku ken adalah Sn = 73 n3 + 32 n2 +

3.2

67
n.
6

Arithmetic, Geometric, Harmonic, and Quadratic


Means
Rataan Aritmatik, Geometrik, Harmonik dan
Kuadratik

In mathematics or statistics, an arithmetic mean of a list of numbers is the sum


of all of the list divided by the number of items in the list. The arithmetic mean
is the most commonly-used type of average and is often referred to simply as
the average. A geometric mean is a type of mean or average, which indicates
a central tendency of numbers. It is similar to arithmetic mean, which is what
most people think of with the word average, except that instead of adding the
set of numbers and then dividing the sum by the count of numbers in the set
n, the numbers are multiplied and then the nth root of the resulting product is
taken. A harmonic mean (formerly sometimes called the subcontrary mean) is
one of several kinds of average. Typically, it is appropriate for situations when
the average of rates is desired. A quadratic mean is a type of average which is
calculated as the square root of the mean of the squares.
Dalam matematika atau statistika, suatu rataan aritmatik dari daftar bilangan
adalah jumlah bilangan itu dibagi dengan banyaknya bilangan dalam daftar.
Rataan aritmatika adalah sesuatu yang paling sering dipakai dan secara umum
juga disebut dengan rata-rata. Rataan geometrik adalah jenis rataan atau ratarata yang merepresentasikan suatu tendensi sentral dari sekumpulan bilangan.
Hampir sama dengan rataan aritmatik dimana banyak orang berpikir bahwa ini

44

Chapter 3. Algebra Functions

hanyalah rata-rata, namun dalam rata-rata seluruh bilangan dalam daftar dijumlahkan kemudian dibagi dengan banyaknya bilangan dalam daftar, untuk
rataan geometrik semua bilangan dalam daftar dikalikan kemudian ditarik akar
pangkat n. Rataan harmonik (dulu disebut sebagai kebalikan rataan) adalah
jenis lain dari rata-rata. Biasanya ini sesuai untuk menentukan rata-rata yang
mempertimbangkan tingkatan. Rataan kuadratik adalah tipe dari rata-rata yang
diperoleh dari menarik akar kuadrat dari rata-rata kuadratnya.
Let AM =Arithmetic Mean, GM =Geometric Mean, HM =Harmonic Mean and
QM =Quadratic Mean. For any positive real numbers a, b, it satisfies
Misal AM =Rataan Aritmatik, GM =Rataan Geometrik, HM =Rataan Harmonik
and QM =Rataan Kuadratik. Untuk sebarang a, b bilangan real positif, berlaku
(a b)2 0 a2 + b2 2ab

(3.13)

a and b b then, from (3.13), it follows a + b 2 ab or

Jika a a dan b b maka (3.13) menjadi a + b 2 ab atau


If a

a+b
ab
2

(3.14)

In general we can present (3.14) as follows


Secara umum kita dapat menulis (3.14) sebagai

x1 + x2 + + xn
n x1 x2 . . . x n
{z
}
n
{z
} |
|
GM

AM

If a

1
a

Jika a

and b
1
a

1
b

dan b

(3.15)

then (3.14) shows


1
b

maka (3.14) menjadi

ab

1
a

2
+

1
b

(3.16)

45

Chapter 3. Algebra Functions


In general, we can present (3.16) as
Secara umum kita dapat menulis (3.16) sebagai

x1 x2 . . . x n

1
x1

1
x2

n
+ + x1n
{z
}

(3.17)

HM

From (3.13), we get 2(a2 + b2 ) a2 + 2ab + b2 2(a2 + b2 ) (a + b)2 . If the two


2
2
2
sides are divided by 4 then we have a +b
a+b
, or
2
2
Dari (3.13) didapat 2(a2 + b2 ) a2 + 2ab + b2 2(a2 + b2 ) (a + b)2 . Bila kedua
a+b 2
2
2
ruas dibagi 4, diperoleh a +b

, atau
2
2
r

a2 + b2
a+b

2
2

(3.18)

In general (3.18) can be written as


Secara umum (3.18) ditulis sebagai
r
x21 + x22 + + x2n
x1 + x2 + + xn

n
{zn
}
|

(3.19)

QM

Therefore, from(3.15), (3.17) and (3.19) we can conclude that for any positive real
numbers x1 , x2 , . . . , xn satisfy:
Dengan demikian, berdasarkan (3.15), (3.17) dan (3.19) dapat disimpulkan bahwa
untuk setiap bilangan real positif x1 , x2 , . . . , xn berlaku:
QM AM GM HM
where

46

Chapter 3. Algebra Functions


dimana
r

x21 + x22 + + x2n


n
x1 + x2 + + xn
AM =
n

GM = n x1 x2 . . . xn
n
HM = 1
1
+ x2 + + x1n
x1
QM =

Example. Let x, y, z be any real positive numbers such that x + y + z = 1. Prove


that xy(x + y)2 + yz(y + z)2 + xz(x + z)2 4xyz.
Contoh. Diketahui x, y, z adalah bilangan real positif sehingga x + y + z = 1.
Buktikan bahwa xy(x + y)2 + yz(y + z)2 + xz(x + z)2 4xyz.
Solution. Since x + y + z = 1, we have
Solusi. Karena x + y + z = 1 maka didapat
xy(1 z)2 + yz(1 x)2 + xz(1 y)2 4xyz
xy + yz + xz 6xyz + xyz 2 + x2 yz + xy 2 z 4xyz
xy + yz + xz + xyz 2 + x2 yz + xy 2 z
1 1 1
+ + +x+y+z
x y z
1 1 1
+ +
x y z
1
3
x+y+z

3 }
| {z
AM

10xyz
10
9

3
1
x

1
y

1
z

1
z

3
1
x

1
y
{z

HM

Chapter 3. Algebra Functions

3.3

47

The Polynomials and Remainder Theorem


Suku Banyak dan Teorema Sisa

A polynomial f (x) of degree n can be presented as:


Suatu suku banyak f (x) yang berderajad n dinyatakan dengan:
f (x) = a0 xn + a1 xn1 + + an1 x + an
where a0 , a1 , . . . , an are constant, a0 6= 0 and n is a cardinal number.
dimana a0 , a1 , . . . , an adalah konstanta, a0 6= 0 dan n adalah bilangan cacah.

3.3.1

Polynomials Division
Pembagian Suku Banyak

The division of polynomial is similar to the division of numbers. For instance in


the number: Since 3 4 = 12, it follows 12 : 4 = 3 or 12 : 3 = 4. In the case of
12 : 4 = 3, numbers 4, 3 are respectively called divisor and quotient.
Pembagian suku banyak menyerupai pembagian bilangan. Sebagai contoh pada
bilangan: Karena 34 = 12 maka 12 : 4 = 3 atau 12 : 3 = 4. Pada kasus 12 : 4 = 3
maka bilangan 4, 3 masing-masing disebut pembagi dan hasil bagi.
A polynomial f (x) divided by a divisor P (x) will give a quotient H(x) and a
remainder S(x). Mathematically, we can write as:
f (x) = P (x)H(x) + S(x)
where: f (x) = is a polynomial of degree n; P (x) = is a polynomial of degree k;
H(x) = is a polynomial of degree n k; S(x) = is a polynomial of degree k 1.

Chapter 3. Algebra Functions

48

Suatu suku banyak f (x) yang dibagi dengan pembagi P (x) akan menghasilkan
hasil bagi H(x) dan sisanya S(x). Secara matematis dapat ditulis sebagai:
f (x) = P (x)H(x) + S(x)
dimana: f (x) = suku banyak berderajad n; P (x) = suku banyak berderajad k;
H(x) = suku banyak berderajad n k; S(x) = suku banyak berderajad k 1.
When dividing polynomials, we need to consider the followings:
1. If the divisor is a linear term then the quotient and remainder can be obtained by Horner technique.
2. If the divisor is not linear and not be able to be factorized into product of
linear terms then the quotient and remainder can be obtained by identity
technique.
Dalam melakukan pembagian terhadap suku banyak perlu diperhatikan hal-hal
berikut:
1. Jika pembaginya linier, maka hasil bagi dan sisanya dapat dicari dengan
menggunakan cara Horner.
2. Jika pembaginya bukan linier dan tidak dapat diuraikan menjadi bentuk
perpangkatan linier maka hasil bagi dan sisanya dapat dicari dengan pergunakan metoda Identitas.

49

Chapter 3. Algebra Functions

3.3.2

Remainder Theorem
Teorema Sisa

1. If polynomial f (x) is divided by (x a) then the remainder is f (a).


2. If polynomial f (x) is divided by (ax b) then the remainder is f ( ab ).
3. If (x a)|f (x) then f (a) = 0.

1. Jika suatu suku banyak f (x) dibagi dengan (x a) maka sisanya f (a).
2. Jika suatu suku banyak f (x) dibagi dengan (ax b) maka sisanya f ( ab ).
3. Jika (x a)|f (x) maka f (a) = 0.

3.3.3

Factor Theorem
Teorema faktor

1. If (x a) is a factor of f (x) then the root of f (x) = 0 is x = a.


2. If polynomial f (x) satisfies f (a) = 0, f (b) = 0 and f (c) = 0 then f (x) is
divisible by (x a)(x b)(x c).
3. If f (x) is divided by (x a)(x b) then the remainder is S(x) =

(xa)
f (b) +
(ba)

(xb)
f (a).
(ab)

4. If f (x) is divided by (x a)(x b)(x c) then the remainder is S(x) =


(xa)(xb)
f (c)
(ca)(cb)

(xa)(xc)
f (b)
(ba)(bc)

(xb)(xc)
f (a).
(ab)(ac)

50

Chapter 3. Algebra Functions

1. Jika (x a) adalah faktor dari f (x) maka akar dari f (x) = 0 adalah x = a.
2. Jika pada suku banyak f (x) berlaku f (a) = 0, f (b) = 0 dan f (c) = 0 maka
f (x) habis dibagi (x a)(x b)(x c).
3. Jika f (x) dibagi dengan (x a)(x b) maka sisanya S(x) =

(xa)
f (b)
(ba)

(xb)
f (a).
(ab)

4. Jika f (x) dibagi dengan (xa)(xb)(xc) maka sisanya S(x) =


(xa)(xc)
f (b)
(ba)(bc)

3.3.4

(xa)(xb)
f (c)+
(ca)(cb)

(xb)(xc)
f (a).
(ab)(ac)

Properties of Polynomial Roots


Sifat-Sifat Akar-Akar Suku Banyak

In this section, we consider Vietas Formula. Let si be the sum of the products of
distinct polynomial roots rj of the polynomial equation of degree n
Dalam hal ini akan disajikan penggunaan rumus Vieta. Misal si adalah jumlah dari hasil kali akar-akar polinomial yang berbeda rj dari sebuah polinomial
berderajad n
an xn + an1 xn1 + ... + a1 x + a0 = 0
where the roots are taken i at a time (i.e., si is defined as the symmetric polynomial i (r1 , ..., rn ) for i = 1, ..., n). For example, the first few values of si are
dimana akar-akar itu dihitung sebanyak i dalam suatu proses (atau si didefinisikan sebagai polinomial simetrik i (r1 , ..., rn ) untuk i = 1, ..., n). Sebagai contoh, beberapa nilai si yang pertama adalah
s1 = r1 + r2 + r3 + r4 + . . .
s2 = r1 r2 + r1 r3 + r1 r4 + r2 r3 + . . .
s3 = r1 r2 r3 + r1 r2 r4 + r2 r3 r4 + . . .
..
.

51

Chapter 3. Algebra Functions


and so on. Then Vietas formulas states that
dan seterusnya. Maka rumus Vietas dinyatakan sebagai
si = (1)i

ani
an

The followings are some example of Vietas formula.


Berikut ini adalah beberapa contoh dari rumusan Vieta.
1. For the polynomial ax3 + bx2 + cx + d = 0, we have:
Pada suku banyak ax3 + bx2 + cx + d = 0 berlaku:
1) x1 + x2 + x3 = b/a
2) x1 x2 + x1 x3 + x2 x3 = c/a
3) x1 x2 x3 = d/a
2. For the polynomial ax4 + bx3 + cx2 + dx + e = 0, we have:
Pada suku banyak ax4 + bx3 + cx2 + dx + e = 0 berlaku:
1) x1 + x2 + x3 + x4 = b/a
2) x1 x2 + x1 x3 + x1 x4 + x2 x3 + x2 x4 + x3 x4 = c/a
2) x1 x2 x3 + x1 x2 x4 + x1 x3 x4 + x2 x3 x4 = d/a
3) x1 x2 x3 x4 = e/a
We note that if the degree of polynomial is even then the the value of s is positive
otherwise it is negative.
Dapat dicatat bahwa bila pangkat tertinggi dari polinomial adalah genap maka
nilai s adalah positif jika ganjil maka nilai s adalah negatif.
To find some rational roots of polynomials can be used the following steps:
Untuk menentukan beberapa akar rasional dari suku banyak dapat digunakan
langkah-langkah berikut:

Chapter 3. Algebra Functions

52

1. If the sum of all polynomial coefficients is equal to 0 then x = 1 is one of


the root.
Jika jumlah seluruh koefisien suku banyak sama dengan 0, maka x = 1
merupakan salah satu akarnya.
2. If the sum of the coefficients of odd order and even order are the same then
x = 1 is one of the root.
Jika jumlah koefisien pangkat ganjil dan genap adalah sama, maka x = 1
merupakan salah satu akarnya.
3. If (1) and (2) are not applicable then consider a trial and error technique by
finding factors of the coefficient of the lowest order and substituting into
f (x). Observe whether f (x) = 0 or not.
Jika langkah (1) dan (2) tidak memenuhi, maka gunakan cara coba-coba
yaitu dengan menentukan faktor dari koefisien pangkat terendahnya dan
masukkan ke dalam f (x). Amati apakah f (x) = 0 atau tidak.

P ROBLEMS

AND

S OLUTIONS

S OAL - SOAL DAN P EMBAHASAN


1. A geometric series is presented as :
1
1
+ + 2 + 16 + 128 + 1024
32 4
When we insert two numbers in between any two successive numbers
such that it forms a new geometric series, determine the new ratio and
the number of terms of the new geometric series.
Suatu deret geometri diketahui sebagai berikut:
1
1
+ + 2 + 16 + 128 + 1024
32 4

53

Chapter 3. Algebra Functions

Jika disisipkan dua buah bilangan kedalam dua suku yang berurutan pada
deret geometri ini sedemikian hingga deret itu membentuk deret geometri
baru, tentukan ratio dan banyaknya suku deret geometri baru tersebut.
Solution. We have the following:
Solusi. Diketahui berikut:
1
1
+ + 2 + 16 + 128 + 1024
32 4
So (Sehingga)
Un
Un 1
r = 8
r =

Let k, n be number of inserted numbers and terms. We have k = 2 and


n = 6. The new ratio is:
Misal k, n masing-masing adalah banyaknya bilangan yang disisipkan dan
banyaknya suku sebelum disisikpan, maka diperoleh k = 2 dan n = 6.
Ratio yang baru adalah:
r0 =
r0 =

k+1

r0 = 2

Number of the new terms is


Banyaknya suku yang baru adalah
n0 = n + (n 1)k
n0 = 6 + (6 1).2
n0 = 16

54

Chapter 3. Algebra Functions


The desired geometric series is:
Deret geometri yang dicari adalah:
1
1
1 1 1
+
+ + + + 1 +2 + .... + 265
+ 512} +1024
| {z
32 |16{z 8} 4 |2 {z }
|
{z
}
the inserted numbers (bilangan yang disisipkan)

2. Let a, b and c be any riel positive numbers such that abc = 1. Prove that
1
a3 (b+c)

1
b3 (a+c)

1
c3 (a+b)

32 .

Misal a, b dan c adalah bilangan riil positif sedemikian hingga abc = 1.


Buktikan bahwa

1
a3 (b+c)

1
b3 (a+c)

1
c3 (a+b)

32 .

Solution. Let
S=

1
1
1
+
+
a3 (b + c) b3 (a + c) c3 (a + b)

(3.20)

and a = x1 , b = y1 , c = z1 . Let T = x + y + z. Since abc = 1, we have xyz = 1.


Substituting the new a, b, c we have:
Solusi. Misal
S=
dan a =

1
,
x

b =

1
,
y

1
1
1
+ 3
+ 3
+ c) b (a + c) c (a + b)

a3 (b

c =

1
.
z

(3.21)

Misal T = x + y + z. Karena abc = 1 maka

xyz = 1. Dengan mensubstitusikan a, b, c yang baru didapat:


1
x2
T 2 (T 2 x2 )
T2
=
=
=
T x
a3 (b + c)
T x
T x
T x
1
y2
T 2 (T 2 y 2 )
T2
=
=
=
T y
b3 (a + c)
T y
T y
T y
1
z2
T 2 (T 2 z 2 )
T2
=
=
=
T z
c3 (a + b)
T z
T z
T z
Substituting the above equations into (3.21) we have the following.
Dengan mensubstitusikan semua persamaan di atas ke dalam (3.21) maka

55

Chapter 3. Algebra Functions


diperoleh berikut ini.

1
1
1
2
S = T
+
+
3T (x + y + z)
T x T y T z

1
1
1
2
+
+
4T
= T
T x T y T z

1
1
1
2 3
= T
+
+
4T
3 T x T y T z

= T 3
2

1
T x

1
T y

1
T z

3
{z

4T

AM

2
T 3

3
1

1
T x

1
T y

{z

1
T z

4T

HM

9
T
4T
(T x) + (T y) + (T z)

9
2
T
4T
3T (x + y + z)

9
2
4T
T
2T
9T
T
x+y+z
3 x+y+z
4T = =
=
2
2
2
2 | {z
3 }
2

=
=
=
=

AM

3
3
3
xyz =
2
2

Therefore
Dengan demikian

1
1
1
3
+
+

a3 (b + c) b3 (a + c) c3 (a + b)
2

3. Given that f (x) is a polynomial of degree 2. When f (x) is divided by x + 1,


the remainder is 3; when f (x) is divided by x 3, the remainder is 23; and

56

Chapter 3. Algebra Functions

when f (x) is divided by x 2, the remainder is 15. Find the polynomial


f (x).
Diketahui f (x) adalah sebuah polinomial berderajad 2. Saat f (x) dibagi
dengan x + 1, maka sisanya adalah 3; saat f (x) dibagi dengan x 3, maka
sisanya adalah 23; dan saat f (x) dibagi dengan x 2, maka sisanya adalah
15. Tentukan polinomial f (x) tersebut.
Solution. Let f (x) = qx2 + mx + n. When f (x) is divided by x + 1, the
remainder is s(1) = q(1)2 + m(1) + n = 3; when f (x) is divided by
x 3, the remainder is s(3) = q(3)2 + m(3) + n = 23; and when f (x) is
divided by x 2, the remainder is s(2) = q(2)2 + m(2) + n = 15. Thus, we
have
Misal f (x) = qx2 + mx + n. Saat f (x) dibagi dengan x + 1, maka sisanya
adalah s(1) = q(1)2 + m(1) + n = 3; saat f (x) dibagi dengan x 3,
maka sisanya adalah s(3) = q(3)2 + m(3) + n = 23; dan saat f (x) dibagi
dengan x 2, maka sisanya adalah s(2) = q(2)2 + m(2) + n = 15. Sehingga,
diperoleh
qm+n=3
9q + 3m + n = 23
4q + 2m + n = 15
Solving the three equations, we get q = 1, m = 3 and n = 5.
Dengan menyelesaikan ketiga persamaan di atas, maka didapat q = 1, m =
3 dan n = 5.

CHAPTER 4

Trigonometry
Trigonometri
Trigonometry (from Greek trigonon triangle + metron measure) is a branch
of mathematics that deals with triangles, particularly those plane triangles in
which one angle has 90 degrees (right triangles). Trigonometry deals with relationships between the sides and the angles of triangles and with the trigonometric functions, which describe those relationships.
Trigonometri (berasal dari kata yunani trigonon segitiga + metron mengukur)
adalah suatu cabang matematika yang berkenaan dengan segitiga, khususnya
segitiga dalam bidang dimana satu sudut mempunyai besar sudut 90 derajad
(segitiga siku-siku). Trigonometri berkaitan dengan hubungan anatara sisi-sisi
dan sudut-sudut sebuah segitiga dengan fungsi trigonometri yang menggambarkan sebuah relasi diantaranya.
Trigonometry has applications in both pure mathematics and in applied mathematics, where it is essential in many branches of science and technology. It is
usually taught in secondary schools either as a separate course or as part of a
precalculus course. Trigonometry is informally called trig.
Aplikasi trigonometri dapat ditemukan baik dalam matematika murni maupun
matematika terapan yang keduanya ini sangat berguna pada beberapa cabang
dalam sain dan teknologi. Trigonometri ini biasanya diajarkan di sekolah menengah baik dalam matapelajaran tertentu atau merupakan bagian dari mata pelajaran kalkulus. Secara informal isitilah trigonometri ini disebut dengan trig.

57

58

Chapter 4. Trigonometry

4.1

Trigonometric Function
Fungsi Trigonometri

Consider the following figure, we define the trigonometric functions as : sin =


y
, cos
r

= xr , tg = yx , ctg = yx , sec =

1
,
cos

cossec =

1
.
sin

Perhatikan gambar berikut, maka fungsi-fungsi trigonometri didefinisikan sebagai: sin = yr , cos = xr , tg = yx , ctg = yx , sec =

1
,
cos

cossec =

1
.
sin

r
0

P (x, y)
x

Figure 4.1: The right triangle trigonometric system


Theorem 4.1.1 Pythagoras equality is stated as follow.
Teorema 4.1.1 Kesamaan Pythagoras dinyatakan dalam berikut ini.
1.

sin2 + cos2 = 1, rumus identitas

2.

1 + ctg 2 = cossec 2

3.

tg 2 + 1 = sec2

The values of trigonometric function in every quadrant is described on a system


of quadrant, where 90o < 2 < 180o , 180o < 4 < 270o and 270 < 3 < 360o .
Considering the system of quadrant, the values of trigonometric function for any
angle can be memorized easily. In Quadrant I, all of trigonometric function

59

Chapter 4. Trigonometry

sign are positive, Quadrant II, only sinus is positive, Quadrant III, only tangent
is positive, and Quadrant IV, only cosinus is positive. Therefore, the keyword to
memorize it is ASTC.
Nilai fungsi pada masing-masing kuadran tertera dalam sistem kuadran, dimana 90o < 2 < 180o , 180o < 4 < 270o dan 270 < 3 < 360o . Dengan
memetakan pada sistem kuadran ini maka nilai fungsi trigonometri untuk sebarang sudut dapat dengan mudah diingat tandanya. Pada kuadaran I all
tanda fungsi trigonometri positif, kuadaran II hanya sinus yang positif, kuadaran
III hanya tangent yang positif dan kuadaran IV hanya cosinus yang positif.
Dengan demikian kata kunci untuk mengingatnya adalah ASTC.

Table 4.1: Trigonometric quadrant system


II

sin 2 > 0

all > 0

III

IV

tg 3 > 0 cos 4 > 0

Thus that, understanding the sign, to obtain the values of trigonometric function
we just need to remember its values at the first quadrant. Use the key words
ASTC to obtain the values on the following quadrant. For instance, to determine
sin 150o , we only need to know sin 30o as sin(180o 30o ) = sin 30o . We refer to
second quadrant that sinus is positive which follows that sin 30o =

1
2

Dengan memahami nilai tanda ini maka untuk hal tertentu kita hanya dapat
mengingat nilai nilai fungsi trigonometri pada kuadran pertama, selanjutnya
gunakan kata kunci ASTC untuk menentukan nilai-nilai di kuadran lainnya. Sebagai contoh: untuk menetukan sin 150o , kita cukup mengetahui sin 30o karena

60

Chapter 4. Trigonometry

sin(180o 30o ) = sin 30o . Kita ingat di kuadran kedua bahwa sinus adalah posisitif, sehingga sin 30o =

1
2

We summarize that the values of sinus, cosinus, tangent and cotangent for special angles are as follows.
Nilai sinus, cosinus, tangent dan cotangent untuk sudut-sudut istimewa pada
kuadran pertama adalah sebagai berikut.

Table 4.2: The value of trigonometric functions for special angles


0o

30o

sin

1
2

cos

tg

ctg

1
3
2

1
3
3

45o

1
2
2

1
2
2
1
1

60o 90o

1
3 1
2
1
2

1
3
3

The trigonometric functions in every quadrant does not remain the same. To
easily memorize the change of trigonometric functions of any angles (X o ),
it can be showed as the following system. From Quadrant I IV in clockwise direction, the situations are B(berubah), Tb(tidak berubah), B(berubah),
Tb(tidak berubah) or in short form we have an acronym BTBT. Applying the
acronym ASTC, we easily put sign + or on the resulted values.
Fungsi trigonometri di beberapa kuadran tidak selalu tetap. Untuk mempermudah ingatan kita tentang perubahan nilai fungsi trigonometri dari beberapa
sudut (X o ) dapat disajikan dalam sistem berikut. Dari kuadaran I
IV berlawanan jarum jam, kondisinya adalah B(berubah), Tb(tidak berubah),
B(berubah), Tb(tidak berubah) atau dapat disingkat BTBT. Dengan menerapkan singkatan ASTC kita dapat memberikan tanda + dan pada nilai-nilai itu

61

Chapter 4. Trigonometry
dengan mudah.

Table 4.3: The value of trigonometric functions for any angle (X o )


Tb
o

B
o

sin(180 ) = sin

sin(90 ) = cos

cos(180o ) = cos

cos(90o ) = sin

tg (180o ) = tg

tg (90o ) = ctg

ctg (180o ) = ctg

ctg (90o ) = tg

Tb

sin(270o ) = cos

sin(360o ) = sin

cos(270o ) = sin

cos(360o ) = cos

tg (270o ) = ctg

tg (360o ) = tg

ctg (270o ) = tg

ctg (360o ) = ctg

If an angle is obtained by turning a phase in clockwise direction then the


angle take a place in Quadrant IV. Thus, we know that the values of trigonometric functions for negative angles are exactly the values of trigonometric functions on Quadrant IV. For detail, see the followings.
Bila sudut diperoleh dengan memutar jangka searah jarum jam maka sudut
berada pada kuadarn IV. Dengan demikian dapat dipahami bahwa nilai
fungsi trigonometri pada sudut negatif setara dengan nilai fungsi trigonometri
pada sistem di atas pada kuadran IV. Nilai selengkapnya disajikan dalam tabel
berikut.
Furthermore, we can develop a quadrant system for trigonometric function values for any angle (X o + ). In this case, we generate an acronym TBTB as a
keyword to remember.

62

Chapter 4. Trigonometry

Table 4.4: The value of trigonometric functions for negative angles


sin() = sin(360o ) = sin
cos() = cos(360o ) = cos
tg () = tg (360o ) = tg
ctg () = ctg (360o ) = ctg

Disamping itu, dapat pula digambarkan sistem kuadran untuk menentukan


nilai fungsi trigonometri pada sudut (X o + ). Dalam sistem ini diperoleh
singkatan TBTB sebagai kata kunci untuk mengingat.

Table 4.5: The value of trigonometric functions for any angle (X o + )


B

Tb

sin(360 + ) = sin

cos(90 + ) = sin

cos(360o + ) = cos

tg (90o + ) = ctg

tg (360o + ) = tg

ctg (90o + ) = tg

ctg (360o + ) = ctg

sin(90 + ) = cos

Tb

sin(180o + ) = sin

sin(270o + ) = cos

cos(180o + ) = cos

cos(270o + ) = sin

tg (180o + ) = tg

tg (270o + ) = ctg

ctg (180o + ) = ctg

ctg (270o + ) = tg

63

Chapter 4. Trigonometry
Example.
Contoh.
sin(270o 30o )
!
cos(180o 45o )
sin(270o 30o )
!
cos(180o 45o )

1. Determine the value of


Tentukan nilai dari

sin(270 30 )
Solution. Remember the angle (X o ), ASTC and BTBT, thus cos(180
o 45o ) =
p
o
cos 30
= 3/2.
cos 45o
o

sin(270 30 )
Solusi. Ingat sudut (X o ), ASTC dan BTBT, maka cos(180
o 45o ) =
p
3/2.

cos 30o
cos 45o

tg (360o +60o )
!
cos(180o +90o )
tg (360o +60o )
!
cos(180o +90o )

2. Determine the value of


Tentukan nilai dari

tg (360 +60 )
Solution. Remember the angle (X o +), ASTC and TBTB, thus cos(180
o +90o ) =

tg 60o
== 3.
sin 90o
o

tg (360 +60 )
Solusi. Ingat sudut (X o +), ASTC dan TBTB, maka cos(180
o +90o ) =

3.

4.1.1

tg 60o
sin 90o

Sine and Cosine Rule


Aturan Sinus dan Cosinus

The following figure shows 4ABC and a circle of radius R.


Gambar dibawah ini memperlihatkan 4ABC dan lingkaran dengan jari-jari R.

Theorem 4.1.2 (Sine Rule) The sine rule states:


Theorem 4.1.3 (Aturan sinus) Aturan sinus mengatakan bahwa:
b
c
a
=
=
= 2R
sin
sin
sin

64

Chapter 4. Trigonometry

0
A

(ii)

(i)

(iii)

Figure 4.2: Triangle and circle of radius R


Proof. As an exercise.
Bukti. Sebagai bahan latihan.

Theorem 4.1.4 (Cosine Rule) The cosine rule states:


Teorema 4.1.2 (Aturan cosinus) Aturan cosinus mengatakan bahwa:
a2 = b2 + c2 2bc cos
b2 = a2 + c2 2ac cos
c2 = a2 + b2 2ab cos
Proof. Consider Figure 4.2(iii)
Bukti. Perhatikan Gambar 4.2(iii)
CZ = AC AZ = b c cos
BZ = c sin
BC 2 = CZ 2 + BZ 2
a2 = (b c cos )2 + (c sin )2
= b2 2bc cos + c2 (cos2 + sin2 )
= b2 2bc cos + c2

Chapter 4. Trigonometry

65

Thus a2 = b2 + c2 2bc cos . In the same way, we can prove that b2 = a2 + c2


2ac cos and c2 = a2 + b2 2ab cos .
Sehingga a2 = b2 + c2 2bc cos . Dengan cara yang sama, dapat dibuktikan
b2 = a2 + c2 2ac cos dan c2 = a2 + b2 2ab cos .

Example. Prove that


Contoh. Buktikan bahwa
sin2 = sin2 + sin2 2 sin sin cos
sin2 = sin2 + sin2 2 sin sin cos
sin2 = sin2 + sin2 2 sin sin cos
Solution. The sine rule states that a = 2R sin , b = 2R sin and c = 2R sin .
Substituting into cosine rule above, we will get those formulas.
Solusi. Aturan sinus menyatakan bahwa a = 2R sin , b = 2R sin dan c =
2R sin . Substitusikan ke dalam aturan cosinus di atas maka akan diperoleh
rumus-rumus itu.

4.1.2

Formulas of Sum and Difference of Angles


Rumus-rumus Jumlah dan Selisih Sudut

A circle of radius 1 with point A and B, where AOX = and BOX = , is


presented in Figure 4.3.
Sebuah lingkaran dengan jari-jari 1 dan titik A dan B, dimana AOX = dan
BOX = , terlihat dalam Gambar 4.3.

Theorem 4.1.5 Let and be any angles. The formula of sum and difference of angles
can be stated as follows.

66

Chapter 4. Trigonometry

y
B

A
x

Figure 4.3: Sum and Difference of Angles


Teorema 4.1.3 Diberikan sudut and , maka rumus penjumlahan dan pengurangan
sudut dapat dinyatakan sebagai berikut.
1.

sin( ) = sin cos cos sin

2.

cos( ) = cos cos sin sin


tg tg
tg ( ) =
1 tg tg

3.

Bukti. Consider Figure 4.3, we have A(cos , sin ) and B(cos , sin ). the length
of AB can be obtained by the formula:
Bukti. Perhatikan Gambar 4.3, kita memiliki A(cos , sin ) dan B(cos , sin ).
Panjang AB dapat dicari dengan rumus:
AB 2 = (cos cos )2 + (sin sin )2
= 2(1 (cos cos + sin sin )).

(4.1)

Cosine rule gives


Aturan cosinus memberikan Aturan cosinus memberikan
AB 2 = OA2 + OB 2 2 OA OB cos AOB
= 2(1 cos( )).

(4.2)

67

Chapter 4. Trigonometry
From Equations (4.1) and (4.2), it follows
Dari persamaan (4.1) dan (4.2) memberikan
cos( ) = cos cos + sin sin ,
which completes the proof. We leave other formulas as an exercise

dengan demikian terbuktilah. Rumus lainnya ditinggal untuk bahan latihan. 2

Theorem 4.1.6 Given an angle , the formula of multiple angles is presented as follows.
Teorema 4.1.4 Diberikan sudut , maka rumus rumus sudut rangkap dapat ditunjukkan sebagai berikut.
1.

sin(2) = 2 sin cos

(4.3)

2.

cos(2) = cos2 sin2


2 tg
tg (2) =
1 tg 2

(4.4)

3.

(4.5)

Bukti. For Equation 4.3


Bukti. Untuk Persamaan 4.3
sin(2) = sin( + ) = sin cos cos sin
= 2 sin cos .
We leave others for readers.
Pembuktian lainnya diserahkan kepada pembaca.

Theorem 4.1.7 Let , be any angles, the product formula of trigonometric function
is showed as follows:

Chapter 4. Trigonometry

68

Teorema 4.1.5 Diberikan sebarang sudut and , maka rumus perkalian fungsi trigonometri adalah sebagai berikut:
1.

2 sin cos = sin( + ) + sin( )

2.

2 cos sin = sin( + ) sin( )

3.

2 cos cos = cos( + ) + cos( )

4.

2 sin sin = cos( + ) cos( )

Theorem 4.1.8 Let x, y be any angles, the formula of sum and difference of trigonometric functions are

Theorem 4.1.9 Diberikan sudut x and y, maka rumus penjumlahan dan pengurangan
fungsi trigonometri adalah:
1.
2.
3.
4.

1
1
sin x + sin y = 2 sin (x + y) cos (x y)
2
2
1
1
cos x + cos y = 2 cos (x + y) cos (x y)
2
2
1
1
sin x sin y = 2 cos (x + y) sin (x y)
2
2
1
1
cos x cos y = 2 sin (x + y) sin (x y)
2
2

(4.6)
(4.7)
(4.8)
(4.9)

Proof. Assuming that x = ( + ) and y = ( ), and using Theorem4.1.5, we


can easily prove this theorem.
Bukti. Dengan memisalkan x = ( + ) dan y = ( ), dan menggunakan
Teorema 4.1.5, maka teorema ini dapat dengan mudah dibuktikan.
Example. If 2 cos2 2x + sin( 2 2x) 6 = 0 then cos x = . . .
Contoh. Jika 2 cos2 2x + sin( 2 2x) 6 = 0 maka cos x = . . .

69

Chapter 4. Trigonometry
Solution.
Solution.

2x) 6 = 0
2
2 cos2 2x + cos 2x 6 = 0
2 cos2 2x + sin(

(2 cos2 2x 3)(cos 2x + 2) = 0
3
cos 2x =
cos 2x = 2
2

For cos 2x = 32 2 cos2 x 1 = 32 cos x = 12 5. For cos 2x = 2

2 cos2 x 1 = 2, the value of cos x is undefined. Thus cos x = 12 5 or 12 5.

Untuk cos 2x = 32 2 cos2 x 1 = 32 cos x = 21 5. Untuk cos 2x = 2

2 cos2 x 1 = 2, nilai cos x tak terdefinisi. Sehingga cos x = 21 5 atau 12 5.

4.1.3

Trigonometric Equation
Persamaan Trigonometri

The simple trigonometric equation can be solved by the following way:


Bentuk persamaan trigonometri sederhana dapat diselesaikan dengan cara berikut:

sin x = sin = x1 = + k 360 x2 = (180o ) + k 360


cos x = sin = x1 = + k 360 x2 = + k 360
tg x = sin = x1 = + k 180
ctg x = sin = x1 = + k 180

To solve equation a cos x + b sin x = c, we should transform into equation cos(x


) = y first.
Sedangkan untuk menyelesaikan bentuk persamaan a cos x + b sin x = c, maka

70

Chapter 4. Trigonometry
harus ditransformasikan terlebih dahulu ke dalam bentuk cos(x ) = y.
a cos x + b sin x = c
b
c
cos x + sin x =
a
a
sin
c
b
sin
sin x = , misal tg = =
cos x +
cos
a
a
cos
c
cos x cos + sin sin x = cos
a
c
a
cos(x ) = cos , dimana cos =
a
a2 + b2

(4.10)

So
Sehingga

a cos x + b sin x = c = cos(x ) =

c
a2 +b2

Theorem 4.1.10 Equation of form a cos x + b sin x = c is solvable if c2 a2 + b2

Teorema 4.1.6 Bentuk persamaan a cos x + b sin x = c dapat diselesaikan apabila c2


a 2 + b2
From Equation (4.10), we have a cos x + b sin x = c = cos(x ) =
value of cosine is 1 cos x 1, thus 1
2

c
a2 +b2

1 =

c2

c
.
a2 +b2

a2 +b2

The

1 =

c a + b . Bukti. Sesuai Persamaan (4.10), maka a cos x + b sin x = c =


cos(x ) =
1

c
a2 +b2

c
.
a2 +b2

1 =

Nilai cosinus terletak antara 1 cos x 1, sehingga


c2
a2 +b2

1 = c2 a2 + b2 .

Example. Determine the value of x satisfying tg x + ctg x = 4 cos 2x


Contoh. Tentukan harga x yang memenuhi persamaan tg x + ctg x = 4 cos 2x.

71

Chapter 4. Trigonometry
Solution.
Solusi.
tg x + ctg x = 4 cos 2x
sin2 x + cos2 x
= 4 cos 2x
cos x sin x
1 = 4 cos 2x cos x sin x
sin 90o = 2 cos 2x sin 2x = sin 4x

The last equation shows sin 4x = sin 90o 4x1 = 90o + k 360o 4x1 =
o

(180o 90o ) + k 360o . The value of x satisfying both of them {22 12 , 112 12 ,
o

202 12 , 292 12 }.
Dari persamaan terakhir sin 4x = sin 90o 4x1 = 90o + k 360o 4x1 =
o

(180o 90o ) + k 360o . Nilai x yang memenuhi keduanya adalah {22 21 , 112 12 ,
o

202 12 , 292 12 }.

4.2

Limit Fungsi

In mathematics, the limit of a function is a fundamental concept in calculus and


analysis concerning the behavior of that function near a particular input. Informally, a function assigns an output f (x) to every input x. The function has a
limit L at an input p if f (x) is close to L whenever x is close to p. In other
words, f (x) becomes closer and closer to L as x moves closer and closer to p.
More specifically, when f is applied to each input sufficiently close to p, the result is an output value that is arbitrarily close to L. If the inputs close to p
are taken to values that are very different, the limit is said to not exist. Formal
definitions, first devised in the early 19th century, are given below.
Dalam matematika, limit suatu fungsi adalah merupakan konsep dasar kalkulus
dan analisis terkait dengan karakteristik fungsi di sekitar suatu input tertentu.

72

Chapter 4. Trigonometry

Secara informal, suatu fungsi akan memberikan output f (x) terhadap setiap input x. Fungsi itu akan memberikan sebuah nilai pendekatan L atas suatu input
p jika f (x) dekat dengan L apabila x juga dekat dengan p. Dengan kata
lain, f (x) menjadi dekat dan semakin dekat dengan L sebagaimana x bergerak
dekat dan semakin dekat dengan p. Secara sepesifik, saat f diterapkan terhadap
setiap input yang cukup dekat dengan p, hasilnya adalah sebuah output yang
juga cukup dekat dengan L. Jika input yang dekat dengan p dimasukkan dan
hasilnya sangat berbeda maka limit ini dikatakan tidak terdefinisi. Definisi formal, mucul pertama kali diawal abad 19-an, diberikan dalam berikut ini.

Definition 4.2.1 Intuitive Defintion of Limit. If f (x) is a real function and c is a


real number then:
lim f (x) = L,
xc

that is f (x) can be sufficiently closer to L as x moves close to c.

Definisi 4.2.1 Pengertian limit secara intuitif. Jika f (x) adalah fungsi riil dan c
adalah bilangan riil, maka:
lim f (x) = L,
xc

berarti f (x) dapat dibuat sedekat mungkin dengan L sebagaimana nilai x bergerak dekat
dengan bilangan c.

Theorem 4.2.1 Let f, g be functions whose limits exist for c. Let k, n be respectively
constant and positive real number. We have the followings.

73

Chapter 4. Trigonometry

Teorema 4.2.1 Misal f, g adalah fungsi-fungsi yang limitnya terdefinisi untuk c. Misal
k, n adalah masing-masing sebuah konstanta dan bilangan positif real. Maka akan
berlaku berikut ini.
1.
2.
3.
4.
5.
6.
7.

lim k = k and (dan) lim x = c

xc

xc

lim kf (x) = k lim f (x)

xc

xc

lim[f (x) g(x)] = lim f (x) lim g(x)

xc

xc

xc

lim f (x) g(x) = lim f (x) lim g(x)

xc

xc

xc

limxc f (x)
f (x)
=
, provided (asalkan) lim g(x) 6= 0
xc g(x)
xc
limxc g(x)
h
in
lim[f (x)]n = lim f (x)
xc
xc
q
p
n
lim f (x) = n lim f (x), provided (asalkan) lim f (x) 0
lim

xc

xc

xc

for (untuk) n even (genap)

Definition 4.2.2 Newtons difference quotient. Let f be a function. Function f 0


(read as f prime) is the derivative of f whose value on any x is
f 0 (x) =

f (x + h) f (x)
df (x)
= lim
h0
dx
h

provided the limit exists.

Definition 4.2.3 Hasil beda Newton. Diketahui suatu fungsi f . Fungsi f 0 (dibaca
f aksen) adalah turunan dari f yang nilainya pada sebarang x adalah
f 0 (x) =
asalkan limit ini ada.

f (x + h) f (x)
df (x)
= lim
h0
dx
h

74

Chapter 4. Trigonometry
Example. If f (x) = x3 , then determine f 0 (x).
Contoh. Jika f (x) = x3 , maka tentukan f 0 (x).
Solution.
Solusi.
f (x + h) f (x)
(x + h)3 x3
= lim
h0
h0
h
h
2
2
3
3x h + 3xh + h
= lim
= lim (3x2 + 3xh + h2 )
h0
h0
h
2
= 3x .

f 0 (x) = lim

We expect the readers to remember the formula of derivative function for algebraic, exponential or trigonometry. Expressing the derivative definition is only
to show the connection between derivative concept and the solution of limit of
function. This module does not mean to discuss the concept of derivative in
detail. In the followings, we describe the solution of limit by using a derivative
concept.
Pembaca diharapkan dapat mengingat rumus-rumus turunan fungsi aljabar,
eksponensial dan trigonometri. Pencantuman definisi turunan dalam bagian
ini hanya untuk menggambarkan kaitan konsep turunan dengan penyelesaian
limit fungsi. Sehingga modul ini tidak dimaksudkan untuk membahas konsep
turunan secara detail. Berikut ini diberikan metode penyelesaian limit fungsi
dengan memanfaatkan konsep turunan ini.
In general, if f (x) = xn the the derivative function f 0 (x) is:
Secara umum, jika f (x) = xn maka fungsi turunan f 0 (x) adalah:

f 0 (x) =

d n
x
dx

= nxn1

75

Chapter 4. Trigonometry

4.2.1

Solution Techniques
Metode Penyelesaian

Some techniques in solving limit have been developed. One of the most popular
is by recognizing an undefined form of function y = F (x) when an input x p is
, , 0, 00 , 0 , .
substituted. Those undefined forms are 00 , ,

Beberapa cara menyelesaikan limit fungsi telah dikembangkan, salah satu yang
paling populer yaitu dengan cara mengetahui bentuk takterdefinisi dari funggsi
y = F (x) apabila masukan x p disubstitusikan. Bentuk-bentuk takterdefinisi

itu adalah: 00 , ,
, , 0, 00 , 0 , .

Based on the undefined forms, we can solve the limit of function by the following principles.
Berdasarkan bentuk tak terdefinisi di atas maka dalam menyelesaikan limit fungsi
dapat dilakukan dengan cara berikut.
1. Forms (Bentuk)

0
0

and (dan)

f (x)
f 0 (x)
= lim 0
xc g(x)
xc g (x)

(4.11)

lim

2. Form (Bentuk) 0.
1
f (x)
1
f (x)

lim f (x) g(x) = lim(f (x) g(x))

xc

xc

such that we get forms

0
0

or

= lim

xc

g(x)
1
f (x)

(4.12)

sedemikian hingga diperoleh bentuk

0
0

atau

3. Form Bentuk .
lim[f (x) g(x)] = lim

xc

xc

1
g(x)

1
f (x)

1
f (x)g(x)

(4.13)

76

Chapter 4. Trigonometry
such that we get forms

0
0

or

sedemikian hingga diperoleh bentuk

0
0

atau

4. When we do not end up with the three forms, we can solve it with simplifying the function by using special techniques and mathematics properties,
such as factorization, identity formulas etcetera. For the limit of trigonometric function can be the properties of trigonometry in Chapter 3
Apabila tidak diperoleh ketiga bentuk di atas maka dapat diselesaikan
dengan menyederhanakan fungsi itu dengan menerapkan teknik-teknik
khusus dan sifat-sifat matematis, seperti pemfaktoran, rumus identitas
dan lain sebagainya. Untuk fungsi trigonometri dapat menggunakan sifatsifat trigonometri pada Bab 3.

4.2.2

Limit of Algebraic Function


Limit Fungsi Aljabar

Theorem 4.2.2 Given that a function f (x) = a0 xm + a1 xm1 + a2 xm2 + + am


(x)
is
and g(x) = b0 xn + b1 xn1 + b2 xn2 + + bn . So limx fg(x)
Teorema 4.2.2 Diketahui suatu fungsi f (x) = a0 xm + a1 xm1 + a2 xm2 + + am
(x)
adalah
dan g(x) = b0 xn + b1 xn1 + b2 xn2 + + bn maka limx fg(x)

a0
,
b0

jika
=
0, jika

, jika

m = n,
m < n,
m > n.

(4.14)

Proof. We leave for a group discussion. (Hint. When m = n, divide f (x), g(x)
by xm = xn ; when m < n, divide f (x), g(x) by xn ; and when m > n, divide

Chapter 4. Trigonometry

77

f (x), g(x) by xm ).
Bukti. Sebagai bahan diskusi kelompok. (Petunjuk. Bila m = n maka bagi
f (x), g(x) dengan xm = xn ; bila m < n bagi f (x), g(x) dengan xn ; dan bila m > n
maka bagi f (x), g(x) dengan xm ).

Theorem 4.2.3 Given that a function f (x) = a0 xm + a1 xm1 + a2 xm2 + + am


(x)
and g(x) = b0 xn + b1 xn1 + b2 xn2 + + bn . So limx0 fg(x)
is
Teorema 4.2.3 Diketahui suatu fungsi f (x) = a0 xm + a1 xm1 + a2 xm2 + + am
(x)
dan g(x) = b0 xn + b1 xn1 + b2 xn2 + + bn , maka limx0 fg(x)
adalah

a0

b0 , jika m = n,
=
(4.15)
, jika m < n,

0, jika m > n.

Theorem 4.2.4 Given that a function


p

f (x) = ax2 + bx + c and g(x) = px2 + qx + r.


So limx [f (x) g(x)] is
Teorema 4.2.4 Diketahui suatu fungsi
p

f (x) = ax2 + bx + c dan g(x) = px2 + qx + r,


maka limx [f (x) g(x)] adalah

bq

2a a, jika m = n,
=
,
jika a < p,

+,
jika a > p.

(4.16)

78

Chapter 4. Trigonometry
Proof.
Bukti.

i ax2 + bx + c + ppx2 + qx + r
p
= lim
ax2 + bx + c px2 + qx + r
x
ax2 + bx + c + px2 + qx + r
(b q)x + (c r)
p
= lim
, karena a = p
2
x
ax + bx + c + px2 + qx + r
h

(b q) + (cr)
x
p
b
c
a + x + x2 + p +

lim q

q
x

r
x2

(b q)
a.
2a
h
i
p
Hence limx
ax2 + bx + c px2 + qx + r =
=

(bq)
a.
2a

and a < p, we leave


hfor a group discussion.
i
p
Sehingga limx
ax2 + bx + c px2 + qx + r =

When the case a > p

(bq)
a.
2a

Untuk kasus a > p

dan a < p dapat digunakan sebagai bahan diskusi kelompok.

Apart from the three cases, in general we can solve the limit of algebraic function
by referring the methods in Subsection 4.2.1.
Diluar tiga kasus di atas, secara umum limit fungsi aljabar dapat diselesaikan
dengan mengacu pada metode-metode yang terdapat subbab 4.2.1.
x2 +(3a)x3a
xa
x2 +(3a)x3a
limxa
xa

Example. Determine limxa


Contoh. Tentukan

Solution. Case 4.11, we have


Solution. Kasus 4.11, maka
x2 + (3 a)x 3a
2x + (3 a)
= lim
xa
xa
xa
1
= a+3
lim

x3 +2x2 c+3xc2 5 6c3 5)


xc

x3 +2x2 c+3xc2 5 6c3 5)


limxc
xc

Example. Determine limxc


Contoh. Tentukan

79

Chapter 4. Trigonometry
Solution. Case 4.11, we have
Solusi. Kasus 4.11, maka

x3 + 2x2 c + 3xc2 5 6c3 5)


3x2 + 4xc + 3c2
lim
= lim
xc
xc 2 x3 + 2x2 c + 3xc2 5
xc

c 11
=
2 6c3 5

4.2.3

Limit of Trigonometric Function


Limit Fungsi Trigonometri

Theorem 4.2.5 The followings hold for limit of trigonometry

Teorema 4.2.5 Berikut ini berlaku untuk limit trigonometri


sin x
x
= lim
=1
x0 x
x0 sin x
sin ax
a
lim
=
x0
bx
b
tan ax
a
lim
=
x0
bx
b
sin ax
a
lim
=
x0 tan bx
b
lim

Theorem 4.2.6 The followings hold for limit of trigonometry

(4.17)
(4.18)
(4.19)
(4.20)

80

Chapter 4. Trigonometry
Teorema 4.2.6 Berikut ini berlaku untuk limit trigonometri
sinn x
xn
lim
=
lim
=1
x0 xn
x0 sinn x
an
sinn ax
=
lim
x0 bxn
b
tann ax
an
lim
=
x0
bxn
b
an
sinn ax
lim
= n
x0 tann bx
b

(4.21)
(4.22)
(4.23)
(4.24)

Proof. Case (4.22) can be proved by the following way:


Bukti. Kasus (4.22) dapat dibuktikan dengan cara berikut:
sinn ax
(sin ax)n
=
lim
x0 bxn
x0
bxn

1
sin ax n
=
lim
b x0 x
an
=
, respecting to (berdasarkan) Theorem (Teorema) 4.2.6
b
lim

For the next proof, we leave for an exercise.


Untuk pembuktian selanjutnya diberikan sebagai bahan latihan.

Likely the limit of algebraic function, the method for solving trigonometric function is relatively the same. But, Limit of trigonometric function in general has
undefined output when the input is taken.
Sebagaimana limit fungsi aljabar metode penyelesaian limit fungsi trigonometri
dapat dilakukan dengan cara yang sama. Namun demikian pada umumnya
limit fungsi trigonometri mempunyai bentuk takterdefinisi apabila inputnya digantikan.
sin 8xsin 4x
7x
sin 8xsin 4x
limx0
7x

Example. Determine limx0


Contoh. Tentukan

Chapter 4. Trigonometry

81

Solution. Equation (4.8), so the solution can be obtained in two ways. First, we
should understand the properties of trigonometric function.
Solusi. Persamaan (4.8), maka penyelesaian dapat dilakukan dengan dua cara,
pertama dengan memahami sifat-sifat fungsi trigonometri berikut.
lim

x0

sin 8x sin 4x
2 cos 6x sin 2x
= lim
x0
7x
7x
sin 2x
= lim 2 cos 6x
x0
7x
sin 2x
= lim 2 cos 6x lim
x0
x0 7x
4
=
7

Secondly, we should consider that this problem meets Case (4.11), so


Cara kedua adalah dengan mempertimbangkan bahwa soal ini termasuk kasus
(4.11), sehingga
sin 8x sin 4x
8 cos 8x 4 cos 4x
= lim
x0
x0
7x
7x
4
=
7
lim

(It must be more efficient!)


(Tentu ini lebih efisien!)

P ROBLEMS

AND

S OLUTIONS

S OAL - SOAL DAN P EMBAHASAN


1. Let , and be angles of a triangle. Determine sine and cosine the angles
+ , + and +
Misal , dan adalah sudut-sudut dalam segitiga. Tentukan sinus dan
cosinus dari sudut-sudut + , + dan +

82

Chapter 4. Trigonometry

2. Solution. The sum of all triangle angles is + + = 180o . Consider


the angle (X o ), the acronyms ASTC and BTBT such that sin( + ) =
sin(180o ) = sin . For other problems we leave for exercise.
Solusi. Jumlah seluruh sudut segitiga + + = 180o . Ingat sudut
(X o ), ASTC dan BTBT, maka untuk sin( + ) = sin(180o ) = sin .
Selanjutnya untuk pertanyaan lainnya diberikan sebagai latihan.
sin 5x+sin 3x
= tg 4x
cos 5x+cos 3x
sin 5x+sin 3x
=
bahwa cos
5x+cos 3x

3. Prove that
Buktikan

tg 4x

Solution. Ordinary way. From the formula above:


Solusi. Cara biasa. Sesuai dengan rumus di atas:
1
1
sin A + sin B = 2 sin (A + B) cos (A B)
2
2
1
1
cos A + cos B = 2 cos (A + B) cos (A B)
2
2
Hence, we get
Sehingga diperoleh:
1
1
sin 5x + sin 3x = 2 sin (5x + 3x) cos (5x 3x)
2
2
= 2 sin 4x cos x
1
1
cos 5x + cos 3x = 2 cos (5x + 3x) cos (5x 3x)
2
2
= 2 cos 4x cos x

Therefore
Dengan demikian
2 sin 4x cos x
sin 5x + sin 3x
=
cos 5x + cos 3x
2 cos 4x cos x
= tg 4x

83

Chapter 4. Trigonometry
4. Prove that sin 54o sin 18o =

1
4

Buktikan bahwa sin 54o sin 18o =


Solution. If we multiple by

2 cos 18o
2 cos 18o

1
4

then we will obtain

Solusi. Jika kita mengalikan dengan

2 cos 18o
2 cos 18o

maka kita akan mendapat

(2 sin 18o cos 18o ) sin 45o


sin 36o cos(90o 54o )
=
2 cos 18o
2 sin(90o 18o )
sin 36o cos 36o
sin 36o cos 36o
=
=
2 sin 72o
2 sin(36o + 36o )
o
o
sin 36 cos 36
1
=
=
o
o
2(2 sin 36 cos 36 )
4

sin 45o sin 18o =

1
5. Determine limx1 ( x4 2x+1

1
Tentukan limx1 ( x4 2x+1

1
)
x7 1
1
)
x7 1

Solution. Case category 4.13, thus


Solusi. Kategori kasus 4.13, maka
lim (

x1

1
1
(x4 2x + 1) (x7 1)

)
=
lim
x1 (x4 2x + 1)(x7 1)
x4 2x + 1 x7 1

refer to Case 4.11, we have


Menjadi kategori kasus 4.11, maka
(x4 2x + 1) (x7 1)
(4x3 2 7x6 )
=
lim
x1 (x4 2x + 1)(x7 1)
x1 (4x3 2)(x7 1) + 7(x4 2x + 1)x6
=
lim

5x5 x3 +2x2 x+14


13x5 5
5x5 x3 +2x2 x+14
limx
13x5 5

6. Determine limx
Tentukan

5x5 x3 +2x2 x+14


13x5 5
5x5 x3 +2x2 x+14
limx
13x5 5

Solution. Case 4.14, thus limx

Solusi. Kasus 4.14, maka


q
7 x9 +243x12
7. Determine limx0 3 185x
7x+9x5 +9x12
q
7 x9 +243x12
Tentukan limx0 3 185x
7x+9x5 +9x12

5
13
5
13

84

Chapter 4. Trigonometry
Solution. Case 4.15, applying Theorem 4.2.1, thus we have
Solusi. Kasus 4.15, dengan menerapkan Teorema 4.2.1 maka diperoleh
r
r
7
9
12
18 5x7 x9 + 243x12
3
3 18 5x x + 243x
lim
=
lim
x0
x0
7 x + 9x5 + 9x12
7 x + 9x5 + 9x12
r
3 243
=
=3
9

2
8. Determine limx 16x2 + 6x + 7 16x2 + 10x + 6

Tentukan limx 16x2 + 6x + 7 16x2 + 10x + 6


Solution. Case 4.15, applying Theorem 4.2.1, thus we have
Solusi. Kasus 4.15, dengan menerapkan Teorema 4.2.1 maka diperoleh

2
lim
16x2 5x + 7 16x2 + 10x + 6
x

2
=
lim 16x2 5x + 7 16x2 + 10x + 6
x
6 10 2 1
=
16 =
2 16
4
9. Obtain the value of the following limits
Tentukan harga limit berikut ini:
2x
x0 sin 3x
sin 8x
(iii) lim
x0 tg 5x
(i) lim

tg 5x
x0 6x
3 tg 2x
(iv) lim
x0 sin 12x
(ii) lim

Solution. From Theorem 4.2.6, thus (i) = 23 ; (ii) = 65 ; (iii) = 85 ; (iv) = 12 .


Solution. Sesuai Teorema 4.2.6, maka (i) = 32 ; (ii) = 65 ; (iii) = 85 ; (iv) =
1
.
2
1
5x+x2
1
5x+x2

10. Determine limx0 cossec 4x


Tentukan limx0 cossec 4x
Solution. Case 4.14, thus

85

Chapter 4. Trigonometry
Solusi. Kategori kasus 4.13, maka
lim cossec 4x

x0

1
(5x + x2 ) sin 4x
=
lim
x0 (5x + x2 ) sin 4x
5x + x2
(5 + 2x) cos 4x
= lim
x0 (5 + 2x) sin 4x + 4(5x + x2 ) cos 4x
4
=
=
0

sin3 3xsin3 3x cos 2x


4x2 tg 3 9x
sin3 3xsin3 3x cos 2x
limx0
4x2 tg 3 9x

11. Determine limx0


Tentukan

Solution. Referring Theorem 4.14, thus


Solusi. Mengacu pada Teorema 4.2.6, maka
sin3 3x sin3 3x cos 2x
sin3 3x(1 cos 2x)
=
lim
x0
x0
4x2 tg 3 9x
4x2 tg 3 9x
sin3 3x 2 sin2 x
= lim

x0 4 tg 3 9x
x2
1
=
54
lim

CHAPTER 5

Kombinatorika
Konsep-konsep penting yang perlu dipahami dalam menyelesaikan masalah
dalam kombinatorika diantaranya adalah konsep permutasi dan kombinasi, prinsip inklusi-ekslusi, koefisien binomial, prinsip sarang merpati (pigeon hole principle), paritas dan relasi rekurensi.

5.1

Permutasi dan Kombinasi

Definition 5.1.1 Permutasi. Banyaknya cara menyusun r anggota dari sebuah himpunan yang mempunyai n anggota dengan memperhatikan urutan adalah P (n, r) =
n!
Prn = (nr)!
, dimana n! = n (n 1) . . . 3 2 1. Apabila sebuah objek dapat dipilih lebih dari satu kali maka disebut permutasi dengan pengulangan dengan rumus
P (n, r) = Prn = nr .
Definition 5.1.2 Kombinasi. Banyaknya cara menyusun r anggota dari sebuah himpunan yang mempunyai n anggota tanpa memperhatikan urutan adalah C(n, r) =
n!
Crn = r!(nr)!
. Apabila sebuah objek dapat dipilih lebih dari satu kali maka disebut
kombinasi dengan pengulangan dimana C(n + r 1, r) = Crn+r1 =

(n+r1)!
.
r!(n1)!

Example. Tentukan banyaknya susunan dua angka dari lima angka 1,2,3,4,5.
Solution. Susunan selain diagonal utama adalah suatu permutasi, dan bila termasuk diagonal utama disebut permutasi dengan pengulangan. Sedangkan
susunan kombinasi adalah hanya di atas diagonal utama atau hanya di bawah
diagonal utama.
86

87

Chapter 9. Kombinatorika

1
z}|{
1 |{z}
11

5.2

13

14

15

24

25
35

21

12
z}|{
22
|{z}

31

32

23
z}|{
33
|{z}

41

42

43

34
z}|{
44
|{z}

51

52

53

54

45
z}|{
55
|{z}

Prinsip Inklusi-Ekslusi dan Peluang

Prinsip ini digunakan untuk menentukan kardinalitas dari gabungan himpunanhimpunan yang tidak harus saling lepas.
Theorem 5.2.1 Misal Ai adalah sebarang himpunan, 1 i n. Kardinalitas dari
gabungan himpunan-himpunan yang tidak harus saling lepas adalah:
n
n
X
X
[
\
X

Ai =
|Ai Aj Ak | . . . (1)n+1 Ai
|Ai |
|Ai Aj | +
i=1

i=1

i<j

i<j<k

i=1

Definition 5.2.1 Jika S = {k1 , k2 , . . . , kn } adalah himpunan dari semua kejadian dalam
suatu ruang sampel, sedangkan K adalah kejadian yang terjadi pada himpunan K S
maka peluang terjadinya K adalah p(K) = n(K)
. Dalam hal ini pi 0 dengan
n(S)
i = 1, 2, . . . , n dan p1 + p2 + + pn = 1.
Definition 5.2.2 Misalkan S adalah ruang sampel berhingga dengan kejadian A dan
B, maka probabilitas bersyarat dari kejadian A dengan syarat B, ditulis p(A/B) adalah:
p(A/B) = p(A/B)
.
p(B)
Example. Pada sebuah klub olahraga diketahui bahwa 10 orang menyukai tennis, 15 orang menyukai squash, 12 orang menyukai badminton, 5 orang menyukai

88

Chapter 9. Kombinatorika

tennis dan squash, 4 orang menyukai tennis dan badminton, 3 orang menyukai
squash dan badminton dan 2 orang menyukai ketiga olahraga. Berapa banyak
anggota klub yang menyukai sedikitnya satu dari ketiga cabang olahraga ini?
Solution. Misalkan T, S dan B masing-masing adalah himpunan anggota klub
yang menyukai tennis, squash dan badminton. Maka menurut teorema InklusiEkslusi
|T S B| = |T | + |S| + |B||T S| |T B| |S B| + |T S B|
= 10 + 15 + 12 + 5 43 + 2
= 12
Jadi, banyaknya anggota klub yang menyukai sedikitnya satu dari ketiga cabang olahraga ini adalah 27 orang.
Example. Sepasang dadu dilempar satu kali. Misal A adalah kejadian muncul
mata 2 pada paling sedikit satu dadu, dan B kejadian jumlah mata yang muncul
pada kedua dadu sama dengan 6. Tentukan peluang kejadian A dengan syarat
B.
Solution. S = {11, 12, . . . , 66} sehingga n(S) = 36. A = {21, 22, 23, . . . , 62}, B =
{15, 24, 33, . . . , 51} dan n(A) = 11, n(B) = 5. Dengan demikian A B = {24, 42}
dan n(A B) = 2 sehingga p(A/B) =

5.3

p(A/B)
p(B)

2/36
5/36

= 52 .

Koefisien Binomial

Prinsip ini digunakan untuk menentukan kardinalitas dari gabungan himpunanhimpunan yang tidak harus saling lepas.

89

Chapter 9. Kombinatorika

Theorem 5.3.1 Jika x dan y adalah variabel dan n adalah bilangan asli maka berlaku:
n
X
n nr r
n
(x + y) =
x y
r
r=0
Bila (x + y)n diekspansikan maka berlaku beberapa sifat berikut:
Terdapat (n + 1) suku, jumlah pangkat dari x dan y pada tiap suku adalah
n.
Pangkat dari x turun satu demi satu dari n ke 0, sedangkan pangkat dari y
naik satu demi satu dari 0 ke n.
Koefisien dari suku-suku yang berjarak sama dari xn dan y n adalah sama.
Beberapa contoh dapat dilihat dalam berikut ini.
(x + y)0 = 1
(x + y)1 = x + y
(x + y)2 = x2 + 2xy + y 2
(x + y)3 = x3 + 3x2 y + 3xy 2 + y 3
(x + y)4 = x4 + 4x3 y + 6x2 y 2 + 4xy 3 + y 4
(x + y)5 = x5 + 5x4 y + 10x3 y 2 + 10x2 y 3 + 5xy 4 + y 5
(x + y)6 = x6 + 6x5 y + 15x4 y 2 + 20x3 y 3 + 15x2 y 4 + 6xy 5 + y 6
Kalau koefisien-koefisiennya disusun dalam bentuk segitiga, maka susunan segitiga tersebut dinamakan Segitiga Pascal. Segitiga ini mempunyai sifat: (i) Bilangan pertama dan bilangan terakhir dalam tiap baris adalah 1, (ii) Tiap bilangan selain 1, dapat dicari dengan menjumlahkan dua bilangan tepat di atasnya.
1

90

Chapter 9. Kombinatorika
1+1
1+2+1
1+3+3+1
1+4+6+4+1
1 + 5 + 10 + 10 + 5 + 1
1 + 6 + 15 + 20 + 15 + 6 + 1
Example. Hitunglah berapa nilai dari

P1003 2007
k=0 2k+1 .

Solution. Perhatikan bahwa binomial untuk x = 1 dan y = 1 adalah


(1 + 1)

2007

2007
X
2007
k=0

k
2007

(1 + (1))

2007
X
2007
k=0

2007
X
2007
=
(1)2007k (1)k = 22007
k
k=0
= 22007

(5.1)

2007
X
2007
(1)2007k (1)k = 02007 = 0
=
k
k=0

(1)k = 0

X 2007
X 2007
k
(1) +
(1)k
k
k
k genap
k ganjil

X 2007
X 2007

k
k
k genap
k ganjil
X 2007
k
k genap

2007
X
2007
2k
k=0

= 0
= 0
X 2007
=
k
k ganjil

2007
X
2007
=
2k + 1
k=0

(5.2)

91

Chapter 9. Kombinatorika
Karena menurut (5.2)

2007
X
2007
k=0

=2

2007
X
2007
k=0

2k

2007
X
2007
=2
2k + 1
k=0

Maka sesuai (5.1) diperoleh

2007
2007
X
2007
1 X 2007
1
=
= 22007 = 22006
2k + 1
k
2 k=0
2
k=0
Pn

r = n2n1 .
P
Solution. Menurut koefisien binomial didapat (x+1)n = nr=0 nr xr . Turunkan
Pn n r1
sekali terhadap x didapat n(x + 1)n1 =
. Substitusikan untuk
r=0 r rx
Pn n
Pn n
n1
r1
x = 1 diperoleh n(1 + 1)
= r=0 r r(1)
atau r=0 r r = n2n1 .

P
Example. Buktikan bahwa nr=1 r2 nr = n(n + 1)2n2 .
P
Solution. Menurut koefisien binomial didapat (x+1)n = nr=0 nr xr . Turunkan
P
dua kali terhadap x didapat n(n1)(x+1)n2 = nr=2 nr r(r1)xr2 . Kemudian
Example. Jika n bilangan asli, buktikan bahwa

r=1

substitusikan x = 1 didapat:
n
X
n
n(n 1)(1 + 1)
=
r(r 1)
r
r=2
n
n
X
n 2 X n
n2
n(n 1)2
=
r
r
r
r
r=2
r=2
n
n
X
X
n
n 2
n2
n(n 1)2
+
r=
r
r
r
r=2
r=2
n2

92

Chapter 9. Kombinatorika
Tambahkan n di kedua ruas
n2

n + n(n 1)2

n
X
n

r2

r
r=2

n
n
X
X
n
n 2
n2
n(n 1)2
+
r=
r
r
r
r=1
r=1
n
X
n 2
n2
n1
n(n 1)2
+ n2
=
r
r
r=1
r=2

Sehingga

5.4

Pn
r=1

n
r

r =n+

n
X
n

r2 = n(n + 1)2n2 .

Prinsip Sarang Merpati

Prinsip sarang merpati (pigeon hole principle) juga dikenal dengan nama shoe
box argument atau dirichlet drawer principle.
Theorem 5.4.1 Jika terdapat lebih dari n barang yang didistribusikan ke dalam n buah
kotak, maka sedikitnya satu kotak akan menerima lebih dari satu barang.
Contoh:
Selama bulan juni (30 april) Antony melakukan pertandingan catur sedikitnya
satu kali sehari. Banyak pertandingan selama bulan tersebut tidak lebih dari 45
kali. Tunjukkan bahwa terdapat periode dimana Antony melakukan tepat 14
pertandingan!
Jawab:
Misalkan ai menyatakan banyaknya pertandingan yang dilakukan Antony selama i hari. Karena dalam sebulan banyaknya pertandingan tidak lebih dari 45
kali maka
0 < a1 < a2 < ... < a30 45

Chapter 9. Kombinatorika

93

Jumlahkan dengan 14 didapatkan


14 < a1 + 14 < a2 + 14 < ... < a30 + 14 59
Dari ketaksamaan (1) dan (2) didapatkan 60 bilangan. Karena maksimal hanya
ada 59 bilangan maka setidaknya terdapat i dan j sedemikian hingga ai = aj +
14. Ekivalen dengan ai aj = 14. Terbukti

5.5

Paritas

Prinsip ini digunakan untuk mengeliminasi kemungkinan-kemungkinan tertentu dengan cara memperhatikan dua masalah saja, misalnya ganjil genap atau
hitam putih. Misal kedua bilangan adalah genap atau ganjil maka bilanganbilangan itu mempunyai paritas sama. Jika satu ganjil dan satunya genap maka
bilangan-bilangan itu mempunyai paritas yang berbeda.
Contoh:
Misalkan n > 1 adalah bilangan ganjil dan misalkan pula A adalah matriks
berukuran nxn yang simetris. Jika setiap baris dan kolom S merupakan suatu
permutasi dari 1, 2, ...n Buktikan bahwa setiap i, i = 1, 2, ...n akan muncul pada
diagonal utama!
Jawab:
Karena setiap baris dan kolom adalah permutasi dari 1, 2, 3, ...n akan muncul
sebanyak n kali dalam matriks A. Misalkan submatriks dari A yang terletak
dibawah dan diatas diagonal utama sebagai A1 dan A2 . Andaikan terdapat bilangan i0 diantara 1, 2, 3, ...n yang tidak muncul di A1 dan A2 haruslah sama.
Akibatnya banyaknya i0 yang muncul di A adalah genap, suatu kontradiksi.

Chapter 9. Kombinatorika

5.6

94

Relasi Rekurensi

Definition 5.6.1 Diberikan suatu fungsi numerik a0 , a1 , . . . , ar , . . . untuk sebarang r.


Suatu persamaan yang mengkaitkan ar dengan satu atau lebih ai , untuk i < r, disebut
relasi rekurensi atau persamaan beda (difference equations).
Contoh: Misalkan kita melempar sebuah koin n kali. Berapa banyaknya hasil
percobaan yang didalamnya sisi gambar tidak pernah muncul dua kali berturutturut? Jawab: Misalkan an menyatakan banyaknya hasil percobaan ke-n. Perhatikan bahwa untuk mendapatkan hasil percobaan sehingga sisi gambar tidak
boleh muncul dua kali berturut-turut maka
1. n 1 barisan yang didalamnya sisi gambar tidak pernah muncuk dua kali
bertrut-turut pada barisan ke-n dipasangkan dengan sisi angka.
2. n 2 barisan yang didalamnya sisi gambar tidak pernah muncul dua kali
berturut-turut pada barisab ke-(n 1) dipasankan dengan sisi angka kemudian barisan ke-n dipasangkan dengan sisi gambar.

5.7

Soal-soal dan Pembahasan

Bagian I
1. Berapa cara untuk bergerak diruang xyz dari titk (0, 0, 0) ke (4, 3, 5) sehingga langkah yang diambil adalah arah x positif, y positif dan z positif?
Pembahasan: Karena langkah yang diambil adalah arah x positif,y positif dan z positif, maka langkah dari (0, 0, 0) ke (4, 3, 5) adalah langkah
yang terpendek. Sehingga banyaknya cara untuk bergerak di ruang tersebut yaitu harus terdiri dari 12 langkah dimana 4 langkah ke kanan(x), 3

95

Chapter 9. Kombinatorika

langkah ke belakang y dan 5 langkah ke atas (z). Jadi banyaknya cara


85
adalah 12
= 27720
4
3 5
2. Hitunglah berapa koefesien dari x102 y 98 dari ekspansi (2x 5y)200 !
Pembahasan:
Menurut teorema binomial untuk menghitung koefesien x102 y 98 dari ekspansi
(2x 5y)200 yaitu dengan mengambil r = 102, sehingga didapatkan suku

ke-102, sehingga didapatkan suku ke-102 sebagai berikut 200
(2x)102 (5y)98
102

(2x)102 (5y)98
Jadi koefesien x102 y 98 adalah 200
102
3. Berapa banyak solusi bilangan asli untuk persamaan x1 +x2 +x3 +...+xk =
n?
Pembahasan:
Dibentuk bilangan yi = xi 1. Karena xi merupakan bilangna asli, maka
jelas bahwa yi merupakan bilangan bulat. Sehingga banyak solusi untuk
x1 +x2 +x3 +...+xk = n ekivalen dengan banyak solusi persamaan berikut:
x1 + x2 + x3 + ... + xk = n
(x1 1) + (x2 1) + (x3 1) + ... + (xk 1) = n k
y1 + y2 + y3 + ... + yk = n k
Sehingga banyaknya solusi bilangan bulat adalah

nk+k1
k1

n1
k1

4. Buktikan bahwa diantara n + 1 bilangan bulat yang dipilih terdapat dua


bilangan yan selisihnya hais dibagi n!
Pembahasan:
Misalkan bilangan yang dipilih tersebut adalah a1 , a2 , ..., an+1 . Diambil sebarang bilangan dari a1 , a2 , ..., an+1 misalkan a1 . Dibentuk
a1 a2 ,
a1 a3 ,
...,
a1 an+1

Chapter 9. Kombinatorika

96

Jika salah satu dariselisih diatas habis dibagi n, maka pernyataan terbukti.
jika n selisih diatas tidak habis dibagi n, maka kemudian dibentuk
a1 a2 = t1 n + r1 ,
a1 a3 = t2 n + r2 ,
...,
a1 an+1 = tn n + rn ,
dengan 0 < ri < n. Karena terdapat n hasil bagi sedangkan banyaknya
bilangan 0 < ri < n hanya ada n 1 bilangan, maka pasti terdapat dua bilangan yang mempunyai sisa hasil bagi yang sama. Akibatnya didapatkan
dua bilangan yang selisihnya habis dibagi n. Terbukti.
5. Diberikan himpunan n + 1 bilangan bulat yang kurang dari 2n. Buktikan
bahwa setidaknya terdapat satu anggota himpunan yang membagi habis
anggota himpunan yang lain!
Pembahasan:
Misalkan anggota himpunan tersbut adalah x1 , x2 , x3 , ..., xn+1 . Untuk setiap i, xi = 2nii yi , dengan ni adalah bilangan bulat non-negatif dan yi adalah
bilangan ganjil. T = {yi : i = 1, 2, ..., n + 1}, maka T adalah koleksi n + 1
bilangan ganjil yang kurang dari 2n. Karena hanya terdapat n bilangan
ganjilyang kurang dari 2n, maka setidaknya terdapat dua bilangan yang
sama misalkan yi = yj , i < j. Akibatnya xi = 2ni yi dan xj = 2nj yj . Jika
ni nj maka xj membagi xi dan jika ni nj maka xi membagi xj . Terbukti.
6. Misalkan x1 , x2 , x3 , ..., xn adalah sebarang permutasi dari 1, 2, 3, ...n. Jika
n adalah bilangan ganjil buktikan bahwa perkalian (x1 1)(x2 2)(x3
3)...(xn n) adalah genap!
Pembahasan:
Andaikan hasil perkalian tersebut ganjil, maka tiap-tiap faktor tidak bernilai genap. Jadi (xk k) bernilai ganjil untuk setiap k. Diketahui n bilangan

Chapter 9. Kombinatorika

97

ganjil maka jumlah semua bilangan ganjil tersebut juga ganjil. Sedangkan
(x1 1)(x2 2)(x3 3)...(xn n) = 0 bilangan genap. Kontradiksi, Jadi,
terbukti bahwa hasil perkalian tersebut bernilai genap.
7. Buktikan bahwa ada 2n 2 bilangan yang terdiri dari atas n angka 1 dan 2
dengan masing-masing angka muncul sedikitnya sekali!
Pembahasan:
Karena banyaknya bilangan dengan angka 1 tidak pernah muncul adalah
1. Jadi banyaknya bilangan yang terdiri dari angka 1 dan 2 dengan masingmasin angka muncul sedikitnya sekali adalah 2n 1 1 = 2n 2 Terbukti.
8. Misakan T adalah sebuah segitiga sama sisi dengan panjang 1. Tunjukkan
bahwa jika terdapat 5 titik pada T maka dua diantaranya akan berjarak
tidak lebih dari 12 !
Pembahasan:
Bentuk tiga garis pada segitiga sama sisi tersebut sehingga tiga garis tersebut akan menghasilkan empat buah segitiga kecil sama sisi. Jelas bahwa
maksimal panjang segitiga kecil sama sisi tersebut adalah 12 . karena terdapat lima titik dan empat segitiga kecil, maka pasti terdapat dua titik yang
terletak pada segitiga kecil yang sama. Terbukti.
9. Tentukan banyaknya daerah yang dibentuk oleh n garis pada bidang datar.Pembahasan:
Diasumsikan tidak ada dua garis yang sejajar dan tidak ada tiga garis yang
berpotongan pada titik yang sama. Misalkan an menyatakan banyaknya
daerah yang dicari. Karena garis ke-n dipotong oleh n 1 garis sebelumnya menjadi n ruas garis dan setiap ruas garis memotong suatu daerah
menjadi dua daerah baru maka didapatkan an = an1 + n dengan a0 =
1, a2 = 2, a3 = 4, a4 = 7. Persamaan yang memenuhi syarat ini adalah
an = 1 + 12 n(n + 1)
10. Misalkan dimiliki perangko dengan nilai 5 sen dan 3 sen. Tunjukkan bahwa

Chapter 9. Kombinatorika

98

dapat dibentuk perangko senilai tepat 8 sen atau lebih!Pembahasan:


(a) Perangko senilai 8 sen jelas dapat dibentuk
(b) Perangko senilai 9 sen jelas dibentuk dengan 3 perangko nil ai 3 sen
dan perangko senilai 10 senjelas dibentuk dengan 2 perangko senilai
n > 10 dilakukan dengan cara berikut:
i. jika perangk n sen terdiri dari minimal 3 perangko bernilai 3 sen,
maka jika perangko ini diganti 2 perangko masing-masing bernilai 5 sen maka akan diperoleh perangko bernilai n + 1 sen.
ii. jika perangko k sen terdiri dari minimal 1 perangko bernilai 5
sen, maka jika perangko 5 sen diganti dengan 2 perangko masingmasing bernilai 3 sen akan diperoleh perangko bernilai n + 1 sen.
Bagian II
1. Berapa banyaknya bilangan bulat positif yang merupakan faktor dari 30030?
Pembahasan:
Karena 30030 = 2x3x5x7x11x13, dan hasil kali r bilangan 2, 3, 5, 7, 11, 13, 1
r 6 juga merupakan faktor dari 30030, maka diperoleh banyaknya faktor

P
adalah 6n=0 n6 = 26
2. Misalkan S sebuah himpunan dengan n unsur. Buktikan bahwa banyaknya
himpunan bagian bagi S ada 2n !
Pembahasan:
Banyaknya unsur di dalam suatu himpunan bagian adalah 0 atau 1 atau
2,... atau n. Selain itu banyaknya himpunan bagian yang memiliki r unsur

adalah nr untuk r = 0, 1, 2, ...n. Dengan demikian, banyaknya himpunan

P
bagian ialah nn=0 nr = 2n .Terbukti.
3. Sebuah kata biner yang panjangnya n adalah suatu barisan angka yang
terdiri atas angka 0 atau 1. Berapa banyak kata biner dengan panjang 10

99

Chapter 9. Kombinatorika
yang diawali dengan tiga angka 0 atau diakhiri dengan dua angka 1?
Pembahasan:

Banyak kata biner dengan panjang 10 yang diawali dengan tiga angka 0
atau diakhiri dengan dua angka 1 = (banyak kata biner yang diawali dengan tiga angka 0) + (banyak kata biner yang diakhiri dengan dua angka
1) - (banyak kata biner yang diawalai dengan tiga angka 0 dan diakhiri
dengan dua angka 1). Kemudian, untuk mendapatkan kata biner yang
diawali dengan tiga angka 0 adalah 1.1.1.2.2.2.2.2.2.2 = 27 , diakhiri dengan dua angka 1 adalah 2.2.2.2.2.2.2.2.1.1 = 28 dan diawali tiga angka 0
dan diakhiri dua angka 1 adalah 1.1.1.2.2.2.2.2.1.1 = 25 . Dengan demikian
didapatkan 27 + 28 25 .
4. Hitunglah berapa nilai dari

Pk=0 2007
1003 2k1 !

Pembahasan:
Perhatikan bahwa binomial untuk x = 1 dan y = 1 adalah

k=0
X
2007
2007

= (1 + 1)2007 = 22007

.....(1)
Sedangkan binomial untuk x = 1 dan y = 1 adalah

k=0
X
2007
2007

Sehingga

Akibatnya

(1)2007k (1)k = (1 1)2007 = 02007 = 0

k=0
X
2007
(1)k = 0
k
2007
X 2007
X 2007
k
(1) +
(1)k = 0
k
k
genap
ganjil

100

Chapter 9. Kombinatorika
Ekivale dengan

X 2007 X 2007
+
=0
k
k
genap
ganjil
X 2007
X 2007
=
k
k
genap
ganjil

.....(2)
Sehingga dari (1) dan (2) didapatkan
X

1003
1003
X
2007
2007
=
= 22007
2k + 1
2k + 1
k=0
k=0

1003
X
2007
2
= 22007
2k
+
1
k=0
Diperoleh

1003
X
2007
= 22006
2k
+
1
k=0

5. Buktikan bahwa

Pn

2
r=1 r

n
r

= n(n + 1)2n2 !

Pembahasan:
Menurut koefisien binomial didapatkan (x + 1)n =

Pn
r=0

n
r

xr Jika ruas

kanan dan kiri diturunkan dua kali terhadap x maka didapatkan


n(x + 1)
n(n 1)(x + 1)

n1

n2

=
=

n
X
n
r=1
n
X
r=2

rxr1

n
r(r 1)xr2
r

101

Chapter 9. Kombinatorika
Kemudian subsitusikan x = 1 sehingga didapakan
n
X
n
n2
n(n 1)2
=
r(r 1)
r
r=2
n
n
X
n 2 X n
n2
n(n 1)2
=
r
r
r
r
r=2
r=2
n
n
X
X
n
n 2
n2
n(n 1)2

r =
r
r
r
r=2
r=2
Tambahkan n diruas kanan dan kiri sehingga diperoleh
n
n
X
X
n
n 2
n2
n + n(n 1)2
+
r = n+
r
r
r
r=2
r=2
Dari sini diperoleh
n
X
n

n
X
n 2
n(n 1)2
+
r = n+
r
r
r
r=1
r=2
n
X
n 2
n2
n1
n(n 1)2
+ n2
=
r
r
r=1
n
X
n 2
n2
n(n 1)2
=
r
r
r=1
n2

6. Misalkan terdapat lima titik P1 , P2 , P3 , P4 , danP5 pada bidang dimana masingmasing mempunyai koordinat bilangan bulat. Buktikan bahwa terdapat
sedikitnya sepasang titik (Pi , Pj ) dengan i 6= j sedemikian hingga segmen
garis (Pi , Pj ) memuat sebuah titik Q yang mempunyai koordinat bilangan
bulat dimana Q 6= P atau Q 6= Pj !
Pembahasan:
Berdasarkan ganjil genapnya, terdapat 4 jenis koordinat untuk kelima titik

Chapter 9. Kombinatorika

102

ini yaitu (genap, genap), (ganjil,ganjil), (genap, ganjil) dan (ganjil, genap).
Karena terdapat lima buah titik maka menurut prinsip sangkar merpati,
setidaknya terdapat dua buah titik yang mempunyai jenis yang sama. misalkan kedua titik ini adalah Pi = (xi , yj )danPj = (xj , yj ) dengan i 6= j.
Karena xi da xj paritas yang sama maka
xi + xjadalah genap, demikian

xi +xj yi +yj
juga dengan yi + yj . Akibatnya Q =
, 2
yang merupakan koor2
dinat titik tengah segmen garis mempunyai koordinat bilangan bulat.
7. Misalkan A adalah sebuah himpunan yang beranggotaan 10 bilangan bulat positif yang kurang dari 107. Buktikan bahwa terdapat dua suphimpunan saling lepas dari A yang hasil tamabah dari anggota-anggotanya
adalah sama!
Pembahasan:
Himpunan A yang hasil tambah dari anggotanya terbesar adalah A = dengan hasil tambah sama dengan 1050. Buatlah 1060 kotak dengan label
0 sampai dengan 1050. Sekarang perhatikan sebarang himpunan A dan
semua subhimpunan, tulis hasil tambah anggota-anggota pada selembar
kertas dan masukkan kertas tersebut pada kotak yang sesuai. Terdapat
1024 kertas dan 1060 kotak. Menurut prinsip sangkar merpati maka terdapat sebuah kotak yang berisi sedikitnya dua kertas. Hal ini berarti jumlah
anggota dari dua subhimpunan ini adalah sama. Perhatikan bahwa kedua sub himpunan ini bisa tidak saling lepas. Tetapi jika demikian kita
dapat menghapus anggota sekutu dari dua sub himpunan ini untuk mendapatkan dua sub himpunan lain yang saling lepas dan hasil tambahan
anggota anggotanya sama.
8. Misalkan n > 1 adalah bilangan bulat ganjil, dan misalkan A adalah matriks berukuran nxn yang simetris. Jika setiap baris dan kolom A merupakan suatu permutasi dari 1, 2, ...n, buktikan bahwa setiap i, i = 1, 2, ...n

103

Chapter 9. Kombinatorika
akan muncul pada diagonal utama A!
Pembahasan:

karena setiap baris dan kolom adalah permutasi dari 1, 2, ...n maka setiap
bilangan i, i = 1, 2, ...n akan muncul sebanyak n kali dalam matriks A. Sebut submatriks dari A yang terletak dibawah dan diatas diagonal utama
sebagai A1 dan A2 . Andaikan terdapat bilangan i0 diantara 1, 2, ...n yang
tidak muncul pada diagonal utama A. Karena A simetris maka banyaknya
i0 yang munculnya di A adalah genap. Suatu kontradiksi.
9. Misalkan P 1, P 2, P 3, P 4 dan P 5 terletak didalam sebuah persegi yang panjang sisinya adalah 1. Notasikan dij adalah jarak antara titik Pi dan Pj .
Tunjukkan bahwa terdapat dua titik yang jaraknya kurang dari

2
!
2

Pembahasan:
Bagi persegi tersebut menjadi empat buah persegi kecil yang sama sisi.
Akibatnya menurut pigeon hole setidaknya terdapat dua titik yang terletak pada persegi kecil yang sama. Karena panjang sisi persegi adalah 1,
maka panjang sisi persegi kecil adalah 12 . Menurut teorema phytagoras,
jarak maksimal dari titik sudut ke sudut yang lain yaitu

2
.
2

Terbukti.

10. Berapakah banyaknya bilangan bulat positif diantara 1 dan 1000 yang tidak
habis dibagi 2 dan tidak habis dibagi 5?
Pembahasan:
Misalkan S = {1, 2, 3, ..., 1000}, A = {xS, 2|x} dan {B = {xS, 5|x}. Akan

Chapter 9. Kombinatorika

104

dicari |A0 B 0 |. Menurut hukum De-Morgan, maka didapatkan


|A0 B 0 | = |(A B)0 |
= |S| |(A B)|
= |S| (|A| + |B| |A B|)

1000
1000
1000
= 1000
+

2
5
10
= 400

Jadi banyaknya bilangan bulat positif ada 400 bilangan.


11. Berapakah banyaknya penyelesaian bilangan bulat untuk persamaan x1 +
x2 + x3 = 24 dengan 1 x1 5, 12 x2 18, 1 x3 5?
Pembahasan:
Notasikan y1 = x1 1, y2 = x2 12, y3 = x3 + 1, sehingga persamaan di atas
menjadi y1 + y2 + y3 = 12 dengan penyelesaikan bilangan bulat nonnegatif
dan y1 4, y2 6, y3 13. Misalkan A, B dan C berturut-turut adalah
himpunan semua penyeesaian sedemikian hingga y1 > 4, y2 > 6, y3 >
13. Dengan demikian, banyaknya solusi nonnegatif untuk y1 + y2 + y3 =

31+7
31+5
12 adalah 121+3
=
19,
|A|
=
,
|B|
=
, |C| = 0 dan juga
7
5
T12
T
T
T T
diperoleh |A B| = 1, |A C| = |B C| = |A B C| = 0. Jadi, dari sini
menurut P IE didapatkan 91 (36 + 21 + 0) 0 = 35.
12. Buktikan bahwa banyaknya pemetaan dari M = {1, 2, ...m} ke N = {1, 2, ..., n}
adalah


n
n
m
(n 2)m ... + (1)n1 1m
n
(n 1) +
2
1
m

Pembahasan:
Banyaknya pemetaan dari M ke N adalah nm . Jika Ai adalah himpunan

105

Chapter 9. Kombinatorika

pemetaan dari M ke N sehingga iN tidak mempunyai prapeta, maka


banyaknya pemetaan dari M ke N yang tidak pada adalah |A1 A2 ...
An |.Perhatikan bahwa :

|Ai | = n1 (n 1)m
P
(b) Untuk setiap i1 , i2 N, |Ai1 Ai2 | = (n 2)m . Akibatnya in1 <i2 |Ai1
(a) Untuk setiap iN, |Ai | = (n 1)m . Akibatnya

Pi=1
n

Ai2 | = (n 2)m
Demikian seterusnya sehingga untuk setiap untuk i1 , i2 ...in1 N , akibatnya
Ai1 Ai2 .... Ain | = (n(n 1))m = 1m
Jadi

n
n
n
m
m
n1
|A1 A2 ... An | =
(n 1)
(n 1) ... (1)
1m
1
2
n1
Akibanya banyaknya pemetaan dari M ke N adalah



n
n
n
m
n1
m
m
1m
(n1) ...(1)
(n1)
n |A1 A2 ...An | =
n1
2
1
13. Sebuah pesta dihadiri oleh n orang dan setiap hadirin meninggalkan topinya dibagian penyimpanan. Dalam berapa banyak cara topi-topi itu dikembalikan sehingga tidak seorang pun menerima topinya kembali?
Pembahasan:
Misalkan Ai melambangkan hmpunan kemungkinan hadirin ke-i mendapatkan kembali topinya, untuk setiap i. Banyaknya cara pengembalian
sehingga hadirin ke-i mendapatkan kembali topinya ada (n 1)!. Jadi ,
|Ai = (n 1)!. Selanjutnya untuk setiap pilihan k bilangan bulat berbeda,
misalkan Ai1 i2 i3 ...ik = Ai1 Ai2 Ai3 ... Aik adalah semua himpunan kemungkinan pengembalian sehingga hadirin ke-ik mendapatka kembali topinya. Dari sini dieroleh |Ai1 i2 i3 ...ik | = (n k)!. pencacahan ini tidak bergan-

Chapter 9. Kombinatorika

106

tung pada pemilihan i1 , i2 , i3 ...ik . Karena banyaknya cara mengabilk bi



P
. Selanjutnya
langan adalah nk . sehingga Sk =
|Ai1 i2 i3 ...ik | = nk ! = n!
k!
menurut prinsip inklusi eksklusi diperoleh


n
k n
| Ai | = n!
(n 1) + ... + (1)
.(n k)! + ... + (1)n
1
k
i=1

1
1
1
= n! 1 + + ... +
1! 2!
n!
n
\

14. Letakkan sebuah kuda catur didalam papan catur berukuran 7x7. apakah
mungkin setiap kuda catur secara serempak dapat bergerak menurut aturan yang berlaku?
Pembahasan:
Asumsikan papan catur tersebut diwarnai dengan pola seperti umumnya.
Papan tersebut mempunyai 49 kotak, misalkan 24 diantaranya putih dan
25 warna hitam. Dengan mempertimbangkan kedua catur yang terletak
dalam kotak hitam, jika setiap kuda catur membuat gerakan sesuai aturan,
maka kuda catur tersebut harus bergerak menuju 25 kotak putih. Karena
hanya ada 24 kotak putih yang tersedia, maka ada sebuah gerakan yang
tidak dapat dilakukan. Jadi kuda catur tidak dapat bergerak secara serempak.
15. Letakkan sebuah kuda catur ke dala papan catur berukuran 4xn. Apakah
mungkin kuda catur dapat menempati setiap kotak dan kembali ke kotak
awal dengan gerakan berurutan dalam 4n kali gerakan?
Pembahasan:
Sebelumnya perhatikan untuk kasus papan catur berukuran 7x7. Misalkan
pada awalnya kuda catur terletak dikotak warna hitam. Pada gerakan
pertama, kuda catur akan bergerak menempati kotak berwarna hitam.

Chapter 9. Kombinatorika

107

Demikian seerusnya. Sehingga agar kuda catur dapat kembali menempati posisinya semula dibutuhkan sejumlah gerakan genap. Karena diiginkan kda catur dapat menemati setiap kotak maka dibutuhkan 49 gerakan harus menempati bilangan genap. Jadi dapat disimpulkan bahwa
kuda catur tidak dapat menempati setipa kotak dan kembali ke kotak awal
sesuai dengan aturan.
Selanjutnya untuk kasus diatas, perhatikan bahwa 4xn adalah bilangan
genap. Untuk menanganinya, warnai papan catur dengan pola yang berbeda,
contohnya seperti gambar dibawah. Perhatikan bahwa gerakan kuda catur
dari kotak putih diatas dan dibawah akan mengantarkan kekotak putih
baris kedua dan ketiga. Demikian sebaliknya gerakan dari kotak putih
baris kedua dan ketiga akan mengantarkan ke kotak putih baris pertama
dan keempat. Terdapat n kotak putih di baris pertama dan keempat yang
hanya dapat dicapai dari n kotak putih baris kedua dan ketiga. Hal ini
berakibat jalur kuda catur tidak akan bergerak dari kotak putih ke kotak
hitam. Jadi, kuda catur tidak mungkin dapat menempati setiap kotak dan
kembali ke posisi semula.

16. Misalkan a1 , a2 , ..., a2n+1 adalah himpunan bilangan bulat yang mempunyai sifat jika salah satunya ada yang dipindahkan maka bilangan yang tersisa dapat dibagi dalam dua himpunan bilangan bulat yang jumlahnya
sama. Buktikan bahwa a1 = a2 = ... = a2n+1
Pembahasan:

108

Chapter 9. Kombinatorika

Misalkan A = a1 + a2 + ... + a2n+1 . Dari claim diatas berakibat bahwa


Aai merupakan bilangan genap untuk setiap i. Jadi semua bilangan bulat
a1 , a2 , ..., a2n+1 mempunyai paritas yang sama. Notasikan a adalah bilangan bulat terkecil dari a1 , a2 , ..., a2n+1 . Kemudian dibentuk bi = ai a untuk setiap i. Jelas bahwa b1 , b2 , ..., b2n+1 juga memenuhi sifat diatas. Karena
b1 , b2 , ..., b2n+1 mempunyai paritas yang sama dan salah satu dan salah satunya merupakan bilangan nol, maka paritas dari b1 , b2 , ..., b2n+1 adalah bilangan genap. Andaikan terdapat bilangan diantara b1 , b2 , ..., b2n+1 yang
bukan bilangan nol. Notasikan k adalah bilangan bulat terbesar sedemikian
hingga 2k membagi habis setiap bi . Dibentuk ci =

bi
2k

untuk setiap i. Jelas

bahwa ci , c2 , ..., c2n+1 juga memenuhi sifat diatas. Karena k adalah bilangan
bulat terbesar yang membagi habis setiap bi , maka terdapat ci yang merupakan bilangan ganjil sedangkan salah satu dari ci , c2 , ..., c2n+1 merupakan
bilangan genap. Jadi ci , c2 , ..., c2n+1 tidak mempunyai paritas yang sama.
Kontradiksi dengan yang diketahui bahwa ci , c2 , ..., c2n+1 memenuhi sifat
diatas. Jadi tidak ada bilangan di antara b1 , b2 , ..., b2n+1 yang bukan bilangan nol, sehingga terbukti bahwa a1 = a2 = ... = a2n+1
17. Misalkan an adalah barisan yang terdiri dari n suku dimana suku-suku
1, 2atau3. Jika barisan ini tidak memiliki dua angka 1 yang berurutan, hitunglah berapa banyak barisan a10 !
Pembahasan:
Misalkan Sn menyatakan banyaknya barisan an yang tidak memiliki dua
angka satu yang berurutan. Perhatikan bahwa Sn dapat dibagi menjadi
dua bagian yaitu Sn,1 yaitu banyaknya barisan an yang diawali dengan
angka 1 dan tidak memiliki dua angka 1 dan Sn,2,3 adalah banyaknya barisan
an yang diawali dengan angka 2 dan 3 dan tidak memiliki dua angka 1.
Jelas bahwa Sn,1 diperoleh dengan meletakkan angka 2 atau 3 di awal
unsur-unsur Sn1 .Disamping itu pada setiap unsur di dalam Sn,2,3 angka

109

Chapter 9. Kombinatorika

kedua pasti 1 dan barisan n 2 bersal dari Sn2 . Dari sini diperoleh Sn =
Sn1 + 2Sn2
18. Notasi Qn menyatakan banyaknya cara meletakkan benteng catur pada
papan catur berukuran nxn sedemikian hingga pengaturannya simetri terhadap diagonaldari sudut kiri bawah dengan sudut kanan atas. Buktikan
bahwa Qn = Qn1 + (n 1)Qn2 !
Pembahasan:
Sebuah benteng pada kolom pertama mungkin atau tidak mungkin menduduki kotak pada sudut kiri bawah. Jika iya, maka terdapat Qn1 cara
meletakkan sisa n 1 benteng. Jika tidak, maka benteng dapat diletakkan
di n 1 kotak pada kolom pertama. Peletakan benteng yang pertama
menentukan lokasi kesimettrisannya pada baris pertama. n 2 benteg
sisanya kemudian diletakkan dengan Qn2 benteng sisanya kemudian dapat diletakkan dengan Qn2 cara. Dari sini maka didapatkan Qn = Qn1 +
(n 1)Qn2 .
19. Sebuah koin diundi n kali. Berapakah peluang didapatkan dua kepala
secara berurutan?
Pembahasan:
Notasikan Pn sebagai peluang bahwa dua kepla secara berurutan tidak
muncul dalam n lemparan. Jelas bahwa P1 = 1, P2 =

3
.
4

Untuk n > 2,

maka ada kasus:


(a) Jika lemparan pertama adalah ekor, maka dua ke[ala secara berurutan
tidak muncul di sisa n 1 undian berikutnya dengan peluang Pn1 .
(b) Jika lemparan pertama muncul kepala, maka pada undian kedua harus
kepala untuk menghindari dua kepala muncul berurutan. Kemudian
dua kepala tidak akan muncul pada n1 sisa undian berikutnya dengan peluang Pn2 .

110

Chapter 9. Kombinatorika
Dari sini didapatkan Pn =

1
P
2 n1

+ 41 Pn2 , n > 2 transformasi ini dapat

dirubah kedalam bentuk rekurensi yang lebih umum dengan mengalikan


2n sehingga didapatkan 2n Pn = 2n1 Pn1 + 2n2 Pn2 Misalkan Sn = 2n Pn ,
akan didapatkan Sn = Sn1 + Sn2
20. Misalkan n sebuah bilangan bulat yang lebih besar atau sama dengan 3.
Berikan suatu tafsiran kombinatorial langsung bagi identitas berikut

n+1
=3
4
Pembahasan:
Ambil n titik pada sebuah lingkaran yang berpusat di O. Banyaknya tali

busur yang terbentuk dari titik ini adalah n2 dan banyaknya pasangan
tali busur adalah ... Selanjutnya perhatikan n + 1 titik termasuk n titik
yang diberika dan titik pusat O. Dapat diambil 4 dari n + 1 titik ini untuk

meperoleh n+1
kombinasi. Jika kombinasi demikian ini mengandung O,
4
maka akan diperoleh 6 ruas garis yang 3 diantaranya adalah jari-jari dan 3

lainnya tali busur. Ketiga tali busur ini kemudian menhasilkan 32 pasangan tali busur dengan tepat satu titik sekutu terhadap suatu pasanga yang
dihasilkan. Misalkan {O, A, B, C} menghasilkan tali busur AB, AC, BC
dan tiga pasangan {AB, AC}, {AB, BC} dan {AC, BC}. Selanjutnya su
atu himpunan 4 titik tidak termasuk O menghasilkan 42 tali busur dan
6
pasangan tali busur. Dari 15 pasangan tali busur ini, terdapat 3 pasan2
gan tali busur yang tidak memiliki sekutu. Misalkan {O, A, B, C} menghasilkan tiga pasangan tali busur {AB, AC}, {AB, BC} dan {AC, BC}.

You might also like